Top Banner
1 1) Two pipes A and B fill at A certain rate B is filled at 10,20,40,80,. If 1/16 of B if filled in 17 hours what time it will take to get completely filled Ans 21 2) In a shopping mall with a staff of 5 members the average age is 45 years. After 5 years a person joined them and the average age is again 45 years. What’s the age of 6th person? 3) Find (4x+2y)/ (4x-2y) if x/2y=2 4) Find average speed if a man travels at speed of 24kmph up and 36kmph down at an altitude of 200m. Formula is 2xy/(x+y) 5) Same model as 4th question. But it is on flat surface. Formula is same 2xy/(x+y). 6) Six friends go to pizza corner there are 2 types of pizzas. And six different flavors are there they have to select 2 flavors from 6 flavors. In how many ways we can select?Ans: 6C2 7) 3, 15, x, 51, 53,159,161. Find X Ans: 17 8) 3 friends A, B, C went for week end party to McDonald’s restaurant and there they measure there weights in some order IN 7 rounds. A;B;C;AB;BC;AC;ABC. Final round measure is 155 kg then find the average weight of all the 7 rounds? Ans: 4(155)/7=31 9) There is a toy train that can make 10 musical sounds. It makes 2 musical sounds after being defective. What is the probability that same musical sound would be produced 5 times consecutively? (1 of )? Ans: 1/2 * 1/2 * 1/2 * 1/2 * 1/2 = 1/32 10) (There was a long story, I'll cut short it). There are 5 materials to make a perfume: Lilac, Balsamic, Lemon, and Woody and MI mosaic. To make a perfume that is in demand the following conditions are to be followed: Lilac and Balsamic go together. Woody and MI mosaic go together; Woody and Balsamic never go together. Lemon can be added with any material. (Actually they had also mentioned how much amount of one can be added with how much quantity of the other; but that's not needed for the question.) All of the following combinations are possible to make a perfume except: 1) Balsamic and Lilac 2) Woody and Lemon 3) MI mosaic and Woody 4) MI mosaic and Lilac 11) A triangle is made from a rope. The sides of the triangle are A cm, B cm and C cm (I do not remember the numerical value). What will be the area of the square made from the same rope?Ans: ((A+B+C)/4)2 12) What is the distance of the z-intercept from the x-intercept in the equation ax+by+cz=d (I do not remember the values of a, b, c, d). Ans: sqrt ((d/a) 2+ (d/c) 2) 13) A scientist in Antarctic region conducts research on bears came to know that bears changes according to the location .once he moves 1 mile towards north, then he moves 2 miles towards east, then 1 mile towards south. Now the color of bear he found will be in: Ans: white 14) (1/3) of a number is 3 times more than the (1/6) of the same number?
104

Aptitude Questions

Nov 23, 2014

Download

Documents

lalitkumarsingh

questions related to campus ricrutment..
Welcome message from author
This document is posted to help you gain knowledge. Please leave a comment to let me know what you think about it! Share it to your friends and learn new things together.
Transcript
Page 1: Aptitude Questions

1

 1)  Two pipes A and B fill at A certain rate B is filled at 10,20,40,80,. If 1/16 of B if filled in 17 hours

what time it will take to get completely filled Ans 21 2) In a shopping mall with a staff of 5 members the average age is 45 years. After 5 years a person

joined them and the average age is again 45 years. What’s the age of 6th person? 3) Find (4x+2y)/ (4x-2y) if x/2y=2 4) Find average speed if a man travels at speed of 24kmph up and 36kmph down at an altitude of

200m. Formula is 2xy/(x+y) 5) Same model as 4th question. But it is on flat surface. Formula is same 2xy/(x+y). 6) Six friends go to pizza corner there are 2 types of pizzas. And six different flavors are there they

have to select 2 flavors from 6 flavors. In how many ways we can select?Ans: 6C2 7) 3, 15, x, 51, 53,159,161. Find X Ans: 17 8) 3 friends A, B, C went for week end party to McDonald’s restaurant and there they measure

there weights in some order IN 7 rounds. A;B;C;AB;BC;AC;ABC. Final round measure is 155 kg then find the average weight of all the 7 rounds?

Ans: 4(155)/7=31 9) There is a toy train that can make 10 musical sounds. It makes 2 musical sounds after being

defective. What is the probability that same musical sound would be produced 5 times consecutively? (1 of )?Ans: 1/2 * 1/2 * 1/2 * 1/2 * 1/2 = 1/32

 10) (There was a long story, I'll cut short it). There are 5 materials to make a perfume: Lilac, Balsamic, Lemon, and Woody and MI mosaic. To make a perfume that is in demand the following conditions are to be followed: Lilac and Balsamic go together. Woody and MI mosaic go together; Woody and Balsamic never go together. Lemon can be added with any material. (Actually they had also mentioned how much amount of one can be added with how much quantity of the other; but that's not needed for the question.) All of the following combinations are possible to make a perfume except: 1) Balsamic and Lilac2) Woody and Lemon3) MI mosaic and Woody4) MI mosaic and Lilac

11) A triangle is made from a rope. The sides of the triangle are A cm, B cm and C cm (I do not remember the numerical value). What will be the area of the square made from the same rope?Ans: ((A+B+C)/4)2

 12) What is the distance of the z-intercept from the x-intercept in the equation ax+by+cz=d (I do not remember the values of a, b, c, d).

Ans: sqrt ((d/a) 2+ (d/c) 2) 13) A scientist in Antarctic region conducts research on bears came to know that bears changes

according to the location .once he moves 1 mile towards north, then he moves 2 miles towards east, then 1 mile towards south. Now the color of bear he found will be in:

Ans: white 14) (1/3) of a number is 3 times more than the (1/6) of the same number? Ans is 18 15) There are 11 boys in a family. Youngest child is a boy. What is the probability of all are boys?

a) 2      b) 2!     C) 2048           d) 1024 16) A boy bought a roll A of 56 inches wide and 141 yards long. He also bought B of 77 inches wide

of length 333yards. We don’t want any details of B. Some irrelevant matter. Final question is Time taken for cutting A into 1 yard piece is 2 seconds. Time taken to cut into 141 pieces of 1 yard each is? Ans is 2(141) =242

 17) A Person buys a horse for 15 ponds, after one year he sells it for 20 pounds. After one year,

again he buys the same horse at 30 pounds and sells it for 40 pounds. What is the profit for that person?Ans is 15 pounds

 18) John buys a cycle for 31 dollars and given a cheque of amount 35 dollars. Shop Keeper exchanged the cheque with his neighbor and gave change to John. After 2 days, it is known that cheque is bounced. Shop keeper paid the amount to his neighbor. The cost price of cycle is 19 dollars. What is the profit/loss for shop keeper? Ans is 23(cost price + change given).

Page 2: Aptitude Questions

2

 19) In a family there are some boys and girls. All boys told that they are having equal no of brothers and sisters and girls told that they are having twice the no. of brothers than sisters. How many boys and girls present in a family?

Ans is 4 boys and 3 girls 20) There are certain number of hats and gloves in a box. They are of 41 red, 23 green, 11 orange.

Power gone. But a woman can differentiate between hats and gloves.How many draws are required to obtain a pair of each color.

 21) There is a die with 10 faces. It is not known that fair or not. 2 captains want to toss die for batting selection. What is the possible solution among the following?a) If no. is odd it is head, if no. is even it is tailb)If no. is odd it is tail, if no. is even it is headc) Toss a die until all the 10 digits appear on top face. And if first no. in the sequence is odd then consider it as tail. If it is even consider it as head.I didn’t remembered last option and I don’t know answer.

 22) 2 years ago of A is x times that of B. 3 Years hence the age of A is 4/3 times of B. What is the present age of B in binary form?I didn’t remember the exact values of x and y. You can solve easily.

 23) metal strip of width ‘x’ cm. 2 metal strips are placed one over the other, then the combine length of 2 strips is ‘y’. If ‘z’ strips are placed in that manner. What is the final width of that arrangement?Ans is (z-1) (y-x) +x.

 24) There are 100 men and 100 women on the dance floor. They want to dance with each other. Then which of the following statements is always true:a) There are 2 men who danced with equal no. of women’sb) There are 2 women who danced with equal no. of men

 25) A game is played between 2 players and one player is declared as winner. All the winners from first round are played in second round. All the winners from second round are played in third round and so on. If 8 rounds are played to declare only one player as winner, how many players are played in first round

Ans is 28.

 26) There are 3 boys A, B, C and 2 Girls D, E. D always sit right to A. Girls never sit in extreme

positions and in the middle position. C always sits in the extreme positions. Who is sitting immediate right to E?

Ans is B or C 27) 49 members attended the party. In that 22 are males, 17 are females. The shake hands

between males, females, male and female. Total 12 people given shake hands. How many such kinds of such shake hands are possible?

Ans is 12C2

 28) There are 1000 pillars for a temple. 3 friends Linda, Chelsey, Juli visited that temple. (Some

unrelated stuff) Linda is taller than Chelsea and taller than 2 of 1000 pillars. Julia is shorter than Linda. Find the correct sentence?a) Linda is shorter among themb) Chelsea is taller than Juliac) Chelsea is shorter than Juliad) Cannot determine who is taller among Chelsea and Julia

Ans: d 29) Entry ticket to an exhibition ranges from 1p to 31p. You need to provide exact change at the

counter. You have 31p coin. In how many parts will u divide 31p so that u will provide the exact change required and carry as less coins as possible?a) 22   b) 31   c) 6   d) 32

Ans is 6 30) There are 2 friends Peter and Paul. Peter age is twice as old as Paul when peter was as old as

Paul is now. Sum of the present ages of Peter and Paul is 35.What is the present age of Peter?Ans is 20 

Page 3: Aptitude Questions

3

31)  A lady took out jacket and gloves, which are avialable in blue 26, yellow 30 and red 56. Power goes off, she can distinguish between gloves and jacket but not in colors. What's the possibilty their she will pick up pair of gloves of each color.

 32) Two bowls are taken, one contains water and another contains tea.one spoon of water is added

to second bowl and mixed well, and a spoon of mixture is taken from second bowl and added to the second bowl. Which statement will hold good for the above?(Ans: second liquid in first bowl is smaller than the first mixture in second bowl)

33) Rearrange and categorize the word ‘RAPETEKA’?Ans: bird(parakeet)

34) A lies on mon, tues, wed and speak truths on other days, B lies on Thurs, Fri, Sat and speaks truths on other days. One day a said I lied today and B said I too lied today. What is the day?

35) One grandfather has three grandchildren, two of their age difference is 3, eldest child age is 3 times youngest child’s age and eldest child’s age is two times of sum of other two children. What is the age of eldest child?

Ans: 181) Now pet's age is to times when paul was once. But at that time paul's age=pet's current

age ,how old is pet?

2) Block has 10,9,5 size, how many unit cube is needed to make a block of that size?

3) 23 people are there, they are shaking hands together, how many hand shakes possible, if they are in pair of cyclic sequence.Ans-22

4) 10men and 10 women are there, they dance with each other, is there possibilty that 2 men are dancing with same women and vice versa.Ans-never

5) A lady took out jacket and gloves, which are avialable in blue 26, yellow 30 and red 56. Power goes off, she can distinguish between gloves and jacket but not in colors. What's the possibilty their she will pick up pair of gloves of each color

6) B is taller than j and 3 pillers. P is shorter than B and 2 pillers is j shorter/taller than P?Ans-irrelevant question

7) Sangakara and ponting selects batting by using a dice, but dice is biased so to resolve ponty takes out a coin, what is the probability that dice shows correct option.

8) In school there are some bicycles and 4wheeler wagons. One Tuesday there are 58 wheels in the campus. How many bicycles are there?Ans: 7

9. Two bowls are taken, one contains water and another contains tea.one spoon of water is added to second bowl and mixed well, and a spoon of mixture is taken from second bowl and added to the second bowl. Which statement will hold good for the above?(Ans: second liquid in first bowl is smaller than the first mixture in second bowl)

10. Which is the smallest no divides 2880 and gives a perfect square?a. 1 b.2 c.5 d.6Ans: c

11. Form 8 digit numbers from by using 1, 2,3,4,5 with repetition is allowed and must be divisible by4?a. 31250 b.97656 c.78125 d.97657Ans: c

12. One problem on (a3-b3)/(a2+ab+b2)Ans: ‘a-b’

Page 4: Aptitude Questions

4

13. Rearrange and categorize the word ‘RAPETEKA’?Ans: bird(parakeet)

14. Key words in question (Fibonacci series, infinite series, in the middle of the question one number series is there. I got the series 3 12 7 26 15 b?Ans:54

(Logic: 3*2+1=7 12*2+2=267*2+1=15 26*2+2=54)

15. A father has 7 penny’s with him and 1 water melon is for 1p, 2chickoos for 1p, 3 grapes foe 1p. He has three sons. How can he share the fruits equally?Ans: 1 watermelon, 2chickoos, 1grape

16. A lies on mon, tues, wed and speak truths on other days, B lies on thur, fri, sat and speaks truths on other days.. one day a said I lied today and B said I too lied today. What is the day?

17. Man, Bear, North, South, Walks.Ans: White

18. (1/2) of a number is 3 times more than the (1/6) of the same number?Ans: 9(for any no it can be true)

19. There are two pipes A and B. If A filled 10 liters in hour B can fills 20 liters in same time. Likewise B can fill 10, 20, 40, 80,160. If B filled in (1/16) th of a tank in 3 hours, how much time will it take to fill completely?Ans: 7 hours

20. Keywords: T.Nagar, Chennai, 1-100, prime numbers b/n 140-180, How many 2’s are there?Ans: 20 (Not only 2’s ,1’s,3’s,4’s,5’s,6’s,7’s,8’s,9’s,0’s also 20)

21. One question has last part like difference between two terms is 9 and product of two numbers is 14, what is the squares of sum of numbers?Ans: 109

22. A man is standing before a painting of a man and he says I have no bro and sis and his father is my father’s son?Ans: His son

23. What is the value of [(3x+8Y)/(x-2Y)]; if x/2y=2?Ans: 10 {the numerical may change)

24. A pizza shop made pizzas with to flavours.in home there are ‘N’ different flavors, in that ‘M’ flavors are taken to made pizza.in how many ways they can arrange?(Logic: NcM )

26. In a market 4 man are standing. The average age of the four before 4 years is 45, aftyer some days one man is added and his age is 49. What is the average weight of all?Ans: 49

27. Keywords: One organization, material, labor and maintenance are in the ratio of 4:6:7, the material cost is: 100, what is the total cost?Ans: 425

28. Keywords: Density, Reluctance, Sensitivity, Voltage, Current, what is the Resistance Formula is “R=V/I”

29. Keywords: Sports readers, 10 tables, 4chairs per table, each table has different number of people then how many tables will left without at least one person?

Page 5: Aptitude Questions

5

Ans : 6

30. Keywords: Die, card, coin, b/n 2 to 12Ans: All are equal

31. In a school for a student out of a 100 he got 74 of average for 7 subjects and he got 79 marks in 8th subject. what is the average of all the subjects?The extra 5 marks will be distributed in 8 semester, 5/8=.62574+.625=74.625

32. In a question ,last part has ,the ages of two people has the ratio of 6:5 and by adding the numbers we get 44,after how many years the ratio would be 8:7?Ans: 8

33. Two years before Paul’s age is 2times the Alice age and the present age of Paul is 6times the Alice. What is the presents Paul’s age?( 3years) “You try to solve this question once”

34. One train travels 200m from A to B with 70 km/ph. and returns to A with 80kmph, what is the average of their speed?Just go and hit't

Question 3: The ages of two friends is in the ratio 5:6. After how many years will the ages be in the ratio 7:8?Answer: 10 years.

Question 4: (There was a long story, I'll cut short it). There are 5 materials to make a perfume: Lilac, Balsalmic, Lemon, Woody and Mimosaic. To make a perfume that is in demand the following conditions are to be followed: Lilac and Balsalmic go together. Woody and Mimosaic go together, Woody and Balsalmic never go together. Lemon can be added with any material. (Actually they had also mentioned how much amount of one can be added with how much quantity of the other; but that's not needed for the question.) All of the following combinations are possible to make a perfume EXCEPT:1) Balsalmic and Lilac2) Woody and Lemon3) Mimosaic and Woody4) Mimosaic and LilacAnswer: Mimosaic and Lilac. I have made the question here really easy to understand. But the actual question was in a twisted language and it was difficult to find the answer. It took me some time to get to the answer.

Question 5: A girl has to make pizza with different toppings. There are 8 different toppings. In how many ways can she make pizzas with 2 different toppings.Answer: 8 * 7 = 56

Question 8: An athlete decides to run the same distance in 1/4th less time that she usually took. By how much percent will she have to increase her average speed?Answer: 33.33%

Question 9: A horse chases a pony 3 hours after the pony runs. Horse takes 4 hours to reach the pony. If the average speed of the horse is 35 kmph, what s the average speed of the pony? (This question was really long with loads of irrelevant statement)

1. One man want to build a wall the length and breadth of the wall are 20, 30 respectively. He need 35 bricks for one square centimeter then how many bricks he need?

Ans: l*b*35(no of bricks needed for sqcm) 2. One person had three children. He has 7 pennis. Then how he can distribute the fruits among his

child by folloing conditions.a)he can get one water millon for 1 penny.b)he can get 2 oranges for 1 penny.c)he can get 3 grapes for 1 penny. Ans: 2 water millon 1 orange 1 grape4. In Tnagar many buildings were under residential category.for buildings they number as 1 to 100. For

shops, corporation numbered between 150 and 200 only prime numbers. how many time 6 will

Page 6: Aptitude Questions

6

appear in building numbering?Ans:For 1 to 10 - 1 six    2 to 20 - 1 sixSimilarly upto 59 we utilise six, 5 timesfrom 60 to 69 (including 66) - 11 timesfrom 70 to 100 - 3, hence ans = 5+11+3 = 19Ans:19.

 5. One grand father has 3 grand child. Eldest one age is 3 times of the youngest child age. Sum of two

youngest child age is more than two of eldest one age. Find the eldest one age?Ans: 15(we can easily predict from options, as we take y as 15)6. Difefrence between two numbers is 4 and their product is 17. Then find the sum of their squers?Ans: 70 (By using (x-y)2=x2+y2-2xy)10. One question like. formule?

Ans: Here you can use the (a-b)3=(a-b)(a2+ab+b2)formule). 12. 6 persons standing in queue with different age group, after two years their average age will be 43

and seventh person joined with them. hence the current average age has become 45. Find the age of seventh person?So now we can compute x from above equation. (x = 41, 6x = 246)Let now we compute y,((6x+y)/7) = 45, as we have value of x, compute y.Ans: 69

 13.The ratio between the ages of two pwrsons is 6:5 and sum of there ages is 77 then how many

years later there ratio becomes 8:7?Ans: we can easily predict from options 14. Horse started to chase dog as it relieved stable two hrs ago. And horse started to ran with average

speed 22km/hr, horse crossed 10 mts road and two small pounds with depth 3m, and it crossed two small street with 200 mts length. After traveling 6 hrs, 2hrs after sunset it got dog. compute the speed of dog?Ans: As we have speed and travel time of horse, we can get distance travelled by it.  Hence d = 22*6 = 132km,Exactly this 132km was travelled by dog in 8 hours (as it started two hours earlier).Hence speed of dog = 132/8 = 16.5km/hrAns:16.5km/hr.

 16. 3, 22 , 7, 45, 15, ? , 31

Solution: Here it appear simple, because it arranged in arranged in sequence manner, but the actual question was some what twist mentioning fibonacci series and more over question was in statements (no numbers). Hence first try to understand the question well.Here let group alternate terms 3,7,15,31 (3+4 =7, 7+8 =15, 15+16=31)Similarly for second group (22,45,? (22+23 = 45, 45+46 = 91) hence ans is 91.

 17. Cycles and 4 wheelors problem?Ans: We can easily predict from options 18. Some irrivvelent data.in last two lines problem will be there. One man walks certain distance with 5

kmph and walk back the same  Ans: A 20. 5 friends went for week end party to Mc donalds restrurent  and there they measure there weights.

Some irrilevent data. Final measure is 155 kg then find the average weight of 5 people?Ans: 155/5=31 21. 2 pots are there. 1st pot is filled with ink and 2nd pot is filled with water. Take 1 spoon of ink from

1st pot and pore it in 2nd pot and take 1 spoon of mixture from 2nd pot and pore it in 2nd pot then which one of following is true?

Page 7: Aptitude Questions

7

Ans: Water in 1st pot  is less than the ink in 2nd pot.  23.There r ten spots in library and each spot has 4 tables and ten readers ar there . sorry I don’t

remember complete question?Ans: None 24. Lion and tiger are there. Lion lies on Monday, Tues, Wends and Tiger lies on Thurs, Frid, Sat. Lion

said that today is one of those days when I lies. Tiger said that today is one of those days when I lie too. Then find today?

Ans: Thursday

6: If we subract a number with y, we get 4 increase of number, once it got divided by y itself. Find that number?Ans: 12 (we can easily predict from options, as we take y as 6)

7. I don't remember exactly the question, one logical problem stating the colour of beer?Ans: white. What ever the question about the color of beer means you wrote the answer as white because polar beer. This question is very lengthy don’t burther about that.

8. Jumbled letters, parakeet(answer)Ans: Bird(category)

9. One question like. (209*144)^2 + (209*209)+(209*144)+(144*144) = ?Ans: here you can use calc, many(4 to 5) questions were depend upon calc alone.(no need problem solving technique).

10. I'm only son for my parents. (some irrelevant statements in the middle to distract you). The man in picture is my father's son (some irrelevant statements). Who is he?

A toy train can make 10 sounds sound changes aftr every 4 min, now train is defective and can make only 2 sounds. Find probability that same sound is repeated 3 times consecutively(1 out of)?1. 16  2. 8  3. 12  4. 4Ans:(1/2)*(1/2)*(1/2)=(1/8)Thus 1 out of 8

11. Resistance is X ohm voltage Y then what is currentAns: V=IR

13. Ferrari is leading car manufacturer.*Ferrari S.p.A.* is an Italian sports car. It has enjoyed great success. If Mohan's Ferrari is 3 times faster than his old Mercedes wich gave him 35kmph if Mohan travelled 490 km in his ferrari the hw much time(hours) he took?1. 8  2. 4  3. 7  4. 7.33(Options may be different)

114Lion rat stayin in jungle happily.Lion lies on: Mon, Tue, WedRat lies on:Wed, Thurs SatIf lion says: I didn't lie yesterdayRat says: e1 i didnt lie yesterdaySo what day is today?

14. The ratio of current age of X and Y is 5:7, after hw many years der age ratio will b 7:9?15. Inspired by fibonacci series sanket decided to create is own series which is 1,2,3,7,7,22,15,67, like

this, then what no come immediately before 63?Ans: 202Explanation: Check altenate no.1,3,7,15=====>n*2+1Similarly 2,7,22,67=======>n*3+1So series is 1,2,3,7,7,22,15,67,31,202,63.

 16. By using 1,2,3,4,5, how many 5 digit no. can be formed which is divisible by 4, repetation of no. is

allowed?Ans: Last 2 place should be divisible by 4

Page 8: Aptitude Questions

8

So possible values at last place are 12,24,32,44,52 this can be arranged in 5 waysThe rest 3 places can be filled in 5*5*5 way so total is 5^4

 17. The cost 1 plum is 1 cent, 2 apples is 1 cent, 3 banana is 1 cent. If Rahul buys same amount

of fruits for his 3 sons spending 7 cent then what amount of fruit each child will get?Ans: 1 plum, 2 apple, 1 banana

Explanation: 7/3=2.333 cents for each child according to ans given for d sum each child will get 1 plum, 2 apple, 1 banana 

19. There are to prime no(with some nonsence stuff) then addition of two prime no is 13, and multiplication is 21, then what are some of there squares?Explanation: XY=21 and X+Y=13. Solve using calci

Ans of X & Y will b in points. Then x2+y2=?? 

20. Smita was makin 1 design(again some nonsence) size of larger cube to be made is 5*5*5 using smaller cubes of 1*1*1. She created solid larger cube Then she decided to make hollow cube. Then how many 1*1*1 cubes required to make hollow larger cubeAns= 104

Explanation: (25+25)+(15+15)+(12+12)=104 21. 2X/5Y=5X/3Y Then what is x/y22. A pizza parlor provides pizzas. There were 2 toppings available initially peperoni and salami.

But now they, have introduces 8 new toppings(some names) to select from. A person wishes to buy two different pizzas of new topping in how many ways he can do that?Ans: 8 X 7=56

24. There is a dice having value from 1 6 on each face and a pack of cards having face card aces(hugh chunk of nonsense). When 2 dies are thrown and their scores are added then which sum will come max number of times?1. 8 2. 9 3. 10 4. 11Ans: 8Explanation: 8-2,6  3,5  4,4        9-4,5  3,6       10-5,5  4,6       11-5,6Thus 8's probability is more

25. ''Susha brought terilon cloth and rope to(some nonsence now just go to last 2 lines)''. If rope is 153 meter long and it is to be cut into pieces of 1 meter long then how many times will she have to cut it?Ans: 152 times

26. There are some 2 wheelers and 4 wheelers parked(some nonsense) total number of wheels present is 240 then how many 4 wheelers were there

Ans:This can be done by looking at the option first check the no of bicycles and then multiply it by 2. And

then substract the multiplication value from 240 if the value is divided by 4 then that is the answer

 27. 1/3 of a number is 6 more than 1/6 of that number then what is the number.Ans=x/3=x/6+636 28. The cost of making a robot consists of material cost, repairing cost, coloring cost and is in the ratio

3:4:5. If the material cost is 1200 then find out the cost of the robot.Ans: Simple 3 part is 1200 so 3+4+5=12 part=? 29. There are pepsi 1 litre & oil 1 litre. It is given as 1 spoon of pepsi is aken and is mixed with Oil.

Then 1 spoon oil&pepsi is taken and is mixed with pepsi then which of the condition holds true.Ans: The amount of pepsi in oil is mre than amount of oil in pepsi. 

Page 9: Aptitude Questions

9

30. An tank is filled with water. In first hour 10 lit, in second hour 20 lit and in 3 rd hour time 40 lit. If time taken to fill ¼  of the tank is 5 hour. What is the time required to fill up the tank.

Ans: As the water is filled as twice speed and in 5 the hour ¼. So in 6 hour ½. So answer 7 th hour.he aptitude test consists of 35 questions and here we have been given negative marking of 0.33 per

wrong answer. At the selection the bench mark was 22 marks. It’s an easy test where more than 25 questions can be cleared easily. Never try to solve the questions first that have more number of lines.

 2. Two bowls are taken, one contains water and another contains tea.one spoon of water is added to

second bowl and mixed well, and a spoon of mixture is taken from second bowl and added to the second bowl. Which statement will hold good for the above? 

(Ans: second liquid in first bowl is smaller than the first mixture in second bowl) 3. Form 8 digit numbers from by using 1, 2,3,4,5 with repetition is allowed and must be divisible by4?a.31250    b.97656   c.78125   d.97657  Ans: c  5. One problem on (a3-b3)/(a2+ab+b2) Ans: ‘a-b’ 6. In school there are some bicycles and 4 wheeler wagons. One Tuesday there are 190 wheels in the

campus. How many bicycles are there?Ans: 15  24.   In a school for a student out of a 100 he got 74 of average for 7 subjects and he got 79 marks in

8th

subject. what is the average of all the subjects?Ans: 74.625 25.   In a question ,last part has ,the ages of two people has the ratio of 6:6 and by adding the

numbers weget 44,after how many years the ratio would be 8:7?Ans: 8 26. One train travels 200m from A to B with 70 km/ph. and returns to A with 80kmph, what is the

average oftheir speed? 27.  Two years before Paul’s age is 2times the Alice age and the present age of Paul is 6times the

Alice.What is the presents Paul’s age?( 3years) “You try to solve this question once”2. Two bowls are taken, one contains water and another contains tea. Equal amount . one spoon of

water from 1st is added to second bowl and mixed well, and a spoon of mixture is taken from second bowl and added to the 1st bowl. Which statement will hold good for the above?

{Thought process : Water bowl                                                                tea bowl100                                                                                                 10090w (+10w= spoon vol)                                        100tea+10water90w+ (10*10/11)tea+ 10/11 w                                100t- (10*10/11) t                                                                                        + 10w- 10/11w(1st bowl’s water vol is equal to 2nd bowls tea volume)5. One problem on (785^3-235^3)/(785^2+785*235+235^2)Ans: You are free to carry a calculator with you but you should not use it to solve this kind of problem.

Because it is simple:A3-b3 / a2+ab+b2

Page 10: Aptitude Questions

10

Ans is : a-b here 785-235= 550 thats it.   7. There are two persons paul and Jay .Paul lies on Monday,Tuesday,Wednesday and the remaining

days he speaks truth.Jay lies on Thursday,Friday,Saturday and the remaining Days he speaks truth. once they meet each other, in their conversation Paul says that  yesterday is the day one among those I lie. Jay also says that yesterday  I also lie. what is that day?

a) Sunday.  b) Tuesday.  c) Thursday.  d) Wednesday[Thought process: Now this day cannot be Sunday because in Monday paul speaks truth and Sunday

everyone tells truth. So it must be weekdays. Again, Tuesday  can not possible because Monday and Tuesday paul speaks truth. In case of thrus day, paul speaks lie and Wednesday he speaks true. And joy speaks truth in thrusday and he speak lies in Wednesday.  So, thrusday is the answer.

 21. A question  like a wire is wound whose resistance is  some 132. 44647 ohms. If a voltage of 30

mv, 296 ma current is applied to it what is the resistance.Clue: use formula  R= V/I. beware of voltage & current particularly kilo or milli etc. units.22. In a city,  there are 100 buildings numbered by corporation between 0-100, how many twos are

shown in door,Ans: 2,12,20,21,22,23,24,25,26,27,28,29,32,42,52,62,72,82,92=(20 twos)

 7) Related to A pony & some horse running away from stable. Horse cought the refer after few hours

say 4hours. Speed of horse is 35kmph and he left stable 3 hours after pony left stable then what is speed of pony?

Ans:  pony ran for total 7 hours same distance covered by horse in 4hrs with 35 kmph. Thus distance of horse 35X4 & speed of pony 35X4/7

 8) Mumbai building dept decide to give numbers to building. 1-30 numbers were given to industrial

buildings. 1-100 were given to restaurant buildings. Find how many number of 2's (or 6's or 3's) were used while numbering restaurant buildings.

14) The cost 1 plum is 1 cent ,2 apples   is 1 cent,3 banana is 1 cent,  if A buys same amount of fruits for his 3 sons spending 7 cent den what amount of fruit each child will get? Ans: 1 Plum ,2 Apple,1 Banana

8. There are 10 reading spots in a room. Each reading spot has a round table. Each round table has 4 chair. If different no of persons are sitting at each reading spot. And if there are 10 persons inside the room then how many reading spots donot have atleast a single reader.

(1)  5     (2)   6      (3)  7   (4)  None    Ans 6. because different no of persons are sitting on round table. So possible differnt combinations for

10 people will be 1 2 3 4. because max 4 people can sit on round table. so round tables left are 6. 9. A person do rock climbing at an altitude of 800 m.He go up by 7 mph. and come down by 9 mph.

what was his av speed.Ans (7+9)/2=8. 10. A boys want to make a cuboid of dimension 5m, 6m, 7m. from small cubes of .03 m3. later he

realized he can make same cuboid by making it hollow. Then it take some cubes less. What is the no. of these cube.

Ans. Vol of solid cuboid= 5*6*7=210 m3. Vol of its inner cuboid by removal of which the cuboid will be hollow= (5-2)*(6-2)*(7-2)=60 m3, then ans will be 60/.03

 11. Two years ago A was 6 times older than B. Now he is 2 times older than B. What is the age of A.Ans. Age of A=5, Age of B= Two and half. 12. What is thew value of (78*78*78-45*45*45)/(78*78+78*45+45*45)Ans. 78-45=33.  a3-b3=(a-b)(a2+ab+b2)1. A scientist who is doing a research, found that color of skin of animals vary with change in distance.

One day he started to chase a beer. He walked 1 mile towards north, then turned and walked 1

Page 11: Aptitude Questions

11

mile towards east and then he turned right and caught the beer and radio-ribboned the beer. Then he left the beer at that spot and returned to his base-station taking a turn(walked 1 mile). Then what was the color of beer at that spot?a) White  b) Brown  c) Black   d) Grey

Ans: White(because in North and South pole baears are of white in colour). 2. There is a problem on a race between a horse and a cat, after chasing so much the horse finally

caught the cat when the sun already been set and the moon is in the sky. Some numeric value was given it’s easy just calcute the distance traveled by the horse and by that the speed of that cat. As I remember the Answer: is 66.67 m/sec(check it).

 3. After knowing how the fibbonacci series works two friends of a class tried to invent a new series and

they took 3 and 7 as the first two terms. One day they ask you to say the next term on that series as you are also in the same class. What will be that term.

3,7,7,25,15,  ,31Ans: 61  (here two series are merged 1st:3,7,15,31 and 2nd:7,25,61,) 4. In a colony the municipality decided to number the houses and the shops. They want to number the

shops with the numbers containing 3 between 1-100. The hoses are 10-20 flored and at the back of the house 8 feet is left for fire exit. How many shops can be numbered. 

a.3   b.10   c.19  d.23Ans:19 (Just count the numbers containing 3 between 1-100). 

8. The cost of making robot is divided into cost of material, cost of repairing and cost of painting in the ratio of 5:2:3. The cost used for material is 200. Find the total cost of making the robot?

Ans: 400 (just calculate it). 9. A tank fills in the squence of  10, 20 ,40, 80 (means 10 litre in 1 hr, 20 litr i 2nd nd so). IF at

the 4th hour the tank is 1/4 full. When will it be full?Ans:6 (x/4=80 That means capacity of tank is 320 litres. From the series we can see that in 5th hour it's 160 and then

in 6th hour it's 320)1) Two pipes A and B fill at A certain rate B is filled at 10,20,40,80,. If 1/16 of B if filled in 17 hours

what time it will take to get completely filledAns 212) In a shopping mall with a staff of 5 members the average age is 45 years. After 5 years a person

joined them and the average age is again 45 years. What’s the age of 6th person?

18. a block has 14cm width. if we combine 2 blocks the width will b 25cm. then if we put 150 blocks in similar manner. then wat is the resultant width.

19. a man started at his house and travel 50km towards north and take his left and travel 50km and then take right and travel 50km then in which direction he is from his house

 20. a man has age in years equal to his daughters age in months and daughter age is equals to his

grand daughter age in weeks and her age in weeks total age of 3 members is given we need to find out the mans age

21. problem on data interpretation 22. The cost of making a robot consists of material cost,repairing cost,coloring cost and is in the ratio

3:4:5.if the material cost is 1200 then find out the cost of the robot.23. There are two pipes A and B. If A filled 10 liters in hour B can fills 20 liters in same time. Likewise B

can fill 10,  20, 40, 80,160….if B filled in (1/16) th of a tank in 3 hours, how much time will it take to fill completely?24. In a school for a student out of a 100 he got 74 of average for 7 subjects and he got 79 marks in

8th subject. what is the average of all the subjects?

Page 12: Aptitude Questions

12

 25. Two years before Paul’s age is 2times the Alice age and the present age of Paul is 6times the

Alice. what is the presents Paul’s age???26. 10men and 10 women are there, they dance with each other, is there possibilty that 2 men are

dancing with same women and vice versa. 27. Ferrari is leading car manufacturer.*Ferrari S.p.A.* is an Italian sports car .........it has enjoyed great

success..............If Mohan's Ferrari is 3 times faster than his old MERCEDES wich gave him 35kmph...if Mohan travelled 490 km in his ferrari.........the hw much time he took??

   14) The cost 1 plum is 1 cent ,2 apples is 1 cent,3 banana is 1 cent,  if A buys same amount of fruits

for his 3 sons spending 7 cent den what amount of fruit each child will get? Ans: 1 Plum ,2 Apple,1 Banana

    

2) There was a piece of cloth which is 135 miles long and 35ft wide and some nonsense again then he wants to divide the cloth into pieces with one mile each if he took 7 secs for cutting  piece. What is the time taken for cutting the whole of the cloth into pieces?

135-1=134 x 7=938 secs.4) Problem on ((a3-b3)/(a2-ab-b2))=(a- don waste tym directly calculate it!9) If the area was hit by a virus and so the decrease in the population because of death was x/3 and

the migration from other places increased a population by 2x then annually it had so many ppl. find our the population in the starting.

10) If the milk and water was added in 5:4 ratio then how much more water should be added so he gets the ratio of 6:7.(Not exact question but something like that)

21) 2 buckets with milk and water. A person takes a cup from the bucket with water and pours into the bucket with milk and takes a cup from the milk (mixed with water now) and pours into the bucket with water.

Answer: Both buckets have the same amount of liquid.Answer: If you know the surface area of a cuboid its just simple multiplication.23) Rapunzel walks into the forest of forgetfullness. She meets a Lion who lies on Monday Tuesdays

and Wednesdays and meets a rabbit who lies on Thurs fridays and saturdays . On that day both say that "I lied yesterday". What day is it .

Answer: Easy enough. Try solving it.24) 4 Guys standing in a queue in a supermarket. Their average weight is 43. 1 more guy joins the

queue and the average weight changes to 42. What is the weight of the new guy.Answer: Use formula Avg = sum of weights / Total Number33) In the class of 40 students, 30 speak Hindi and 20 speak English. What is the lowest possible

number of students who speak both the languages?a) 5 b) 20 c) 15 d) 10 e) 3034) If A, B and C are the mechanisms used separately to reduce the wastage of fuel by 30%, 20% and

10%. What will be the fuel economy if they were used combined.35) A can do a piece of work in 20 days, which B can do in 12 days. In 9 days B does Ÿ of the work.

How many days will A take to finish the remaining work? Some Sample questions of TCS. 1) If 20 men take 15 days to complete a job, in how many days can 25 men finish that work?2) One fast typist type some matter in 2hr and another slow typist type the same matter in 3hr. if both

do combine in how much time they will finish.3) The ratio of incomes of C and D is 3:4.the ratio of their expenditures is 4:5.Find the ratio of their

savings if the savings of C is one fourths of his income?4) 6 persons standing in queue with different age group, after two years their average age will be 43

and seventh person joined with them. hence the current average age has become 45. find the age of seventh person?

Page 13: Aptitude Questions

13

Solution: Here the question appear as an easy one, but carried a lot of unwanted sentences and unwanted datas (i dint mention above) in exam which may confuse you on solving technique.

5) Let x be current average age of first 6 persons in queue and current age of seventh person be y. Then 6x will become the sum of those 6 persons age. Now, let compute the sum of those 6 persons after two years, 6x+12 (as each and individual increase their age by 2). hence its average become (6x+12)/6 = 43 (give in question itself). So now we can compute x from above equation. (x = 41, 6x = 246).

 Let now we compute y, ((6x+y)/7) = 45, as we have value of x, compute y. Ans: 697) 3, 22 , 7, 45, 15, ? , 31Solution: Here it appear simple, because it arranged in arranged in sequence manner, but the actual

question was some what twist mentioning fibonacci series and more over question was in statements (no numbers). Hence first try to understand the question well. Here let group alternate terms 3,7,15,31 (3+4 =7, 7+8 =15, 15+16=31)

 Similarly for second group (22,45,? (22+23 = 45, 45+46 = 91) hence ans is 91.8) In Tnagar many buildings were under residential category.for buildings they number as 1 to 100. For

shops, corporation numbered between 150 and 200 only prime numbers. how many time 6 will appear in building numbering?

Ans:For 1 to 10 - 1 six2 to 20 - 1 sixSimilarly upto 59 we utilise six, 5 timesfrom 60 to 69 (including 66) - 11 timesfrom 70 to 100 - 3, hence ans = 5+11+3 = 19Ans:19.10) If we subract a number with y, we get 4 increase of number, once it got divided by y itself.. Find

that number??Ans: 12 (we can easily predict from options, as we take y as 6)14) One question like. (209*144)^2 + (209*209)+(209*144)+(144*144) = ?Ans: Here you can use calc, many(4 to 5) questions were depend upon calc alone.(no need problem

solving technique). 1) Bhaskaracharya lived (some time waste data) sum of two numbers is 5 and product of two numbers

is 8. What is the squares of that numbers?a) 41  b) 13  c) 69  d) 9

Sol: (a-b)^2 =a^2 + b^2 -2aba^2 + b^2 = (a-b)^2 + 2ab=5^2 + 2*8=41.

2) 20 people going to one temple.(some time waste data) They want Rs. 20. For men-Rs.3, women-

Rs.2, children-50 paisa. How much money spend for men, children and women.a) Rs.6-men, Rs.4-children, Rs.10- womenb) Rs.3-men, Rs.7-children, Rs.10- womenc) Rs.3-men, Rs.8-children, Rs.9- womend) Rs.9-men, Rs.3-children, Rs.8- women

Sol: By using options we calculate it easily.From option bMen - 1men*Rs.3= Rs.3Children - 14children* 50paisa= Rs.7Women - 5women*Rs.2= Rs.10

3) Adam sit with his friends in stadium (some time waste data) 5 rounds are running. Every round half

of the teams eliminated. Lastly one team win the game. How many teams are played in the

tournament?a) 30  b)31  c)32  4)33 

Page 14: Aptitude Questions

14

Sol: by using options v can easily find answer.From option c, Total 32 teams played. Next 16,8,4,2,1. In this we choose only even numbers by half next also get even numbers then only we get lastly one

winning team.

4) There are two ipes A and B. If A filled 10 times in hour, B can filled in same time. Likewise B can fill

10,20,40,80.160,. If B filled in 1/16 of the tank in 3 hours, how much time will it take to fill

completely?a) 5hours  b) 7hours  c) 8 hours  d) 10 hours Sol: 1/16th tank - 3rd hour, 1/8th tank – 4th hour, 1/4th tank- 5th hour, ½th tank – 6th hour, Full tank –

7th hour. 5) Shusan brought terlin cloth and rope (some time waste data). If rope is 115metre long and cut 7

times then how many pieces you get?

a) 805  b) 804 c) 802 d) 806Sol: 115*7times =805 pieces.

6) In room glouses, jackets total 11-blue, 46-red, 42-green. In room power is cut (some time waste

data). Probability of take 5 colors of glouse and jackets? (I did not remember but same like

this ).

7) There are some 2 wheelers and 4 wheelers parked(some time waste data).Total number of wheels

present is 140 then how many cycle wheels are there?a) 15 b) 18 c) 19 d) 14

Sol: check optionsFrom a) 15*2=30 cycle wheels, 140-30=110 is not divisible by 4 so 15 wrong ansFrom b) 18*2=36 cycle wheels 140-36=102 is divisible by 4 so 18 is correct ans

8) Average Age of 3 members is X. Two persons added to the 3 members team the average is Y then

find the last person age? (I did not remember the X,Y values).

9) Pizza parlor provides pizzas there were 2 toppings available initially peperomi and salami but now

they introduces 7 new item(some time waste data). One person wishes to buy two different

pizzas from the new items. How many ways he can do that?

Sol: from new items n*(n-1); 7*6=42

12) The ratio of current age of X and Y is 6:5. Their age some days after is 55 years. After how many

years their age ratio will be 8:7?

Sol: 6x:5x6x+5x = 55  x=5;6x:5x=30:25;(30+y)/(25+y)=8/7  y=10

13) A pong ran away from stable to NEW YORK then went to US then to BEJING ran through grass

(Some time wasted data)Horse after 4 hours (some time waste data)then pong went to sky. If

the speed of the horse is 45kmph and he left stable 3 hours then pong’s average speed?

17) Pink’s age before 6 years was “x” times the age of Anderson. Pink’s age after 6 years will be “a/b”

times the age of Anderson. Then finfd the persent age of pink’s ?

Sol: I did not remember the exact values of a,b and x but procedure isP-6 = x(A-6); pink’s age-P, Anderson’s age-AP+6 = (a/b)(A+6)Solve the equations we get Paul’s present age.

18) (some time waste data) 10 boys are there. From that how many ways, we can select the youngest

age person?a) 2 b) 1024 c) 512 d) 2!

Sol: 2^(n-1); 2^(10-1)= 2^9 = 512.

Page 15: Aptitude Questions

15

1) Two tanks A and B. A fills 1 ltr/1 hour B fills 10, 20, 30 per hour. If this is (passage unnecessary). If 1/4th tank of B takes 15 hours to fill how much it time will to take to fill complete tank? 

2) Out of 7 children the youngest is boy then find the probability that all the remaining children are boys

Ans: 1/2^6 = 1/64 12) 4 years before Paul’s age is 3times the Alice age and the present age of Paul is 6times the Alice.

What is the presents Paul’s age?Ans: x-4 = 3(y-4); x=6y: Solve you will get it.16) a volume of A are having in a container of sphere. how many semi hemispheres of  B volume each

will be required to transfer all the A in to semi hemispheres?Ans: A= x B25) Direction problems. A man goes 50 km North, then turned left walked 40 km, then turned right? In

which direction is he in?Ans: North 

 Question 8: An athlete decides to run the same distance in 1/4th less time that she usually took. By

how much percent will she have to increase her average speed?

Answer: 33.33%

  

Question 10: There is 7 friends (A1, A2, A3....A7). If A1 have to have shake with all with out

repeat. How many hand shakes possible?(I dont know the exact question but like this only)Question 11:  There are two pipes A and B. If A filled 10 liters in a hour B can fills 20 liters in same

time. Likewise B can fill 10, 20, 40, 80,160….if B filled in (1/16) th of a tank in 3 hours, how much time will it take to fill completely? Answer: 7 hours

1. There is ferarri and benz car, benz speed is say 10kmph and it cover 10 km.And if ferarri goes with 3 times faster than benz.So in how much time ferarri could take to cover same distance. sol: as speed of ferarri is 3*10=30 so time will be 10/30

2. If one lady have 3 daughter and any of out 3 have diff, of ages is 3.And  oldest is 3 times of more than 2 than yougest after 2 years then tell the age of oldest daughter.Solution: let x is youngest ,y middle ,z oldest. so y-x=3,  z-y=3, and z=2=2(x+2) and put the option  answer  try to get condition.(sorry i forgot option but pattern ll be same)

8.if a and b are mixed in 3:5 ration and b,c are mixed in 8:5 ration if the final mixture is 35 litres,find the amount { a/b=3*8/5*8  and   b/c=8*5/5*5   a/b/c=24:40:25 ans=40*35/(24+40+35)=1400/89

Ans=15.79}of b in the final mixture9.1!+2!+....50!=3*10^64?

 4. MR dash has 3 sons whose ages are respectively a,b,c. The grandfather has bought a cycle for the

eldest son, mother has bought a bag for the youngest one which cost Rs: 150/. The sum of two  age of the elder son & one son is 15. The difference of age of sons is 3 & 2. Then what is the age of the eldest son?

a)10, b)11, c) 12,d)13 5. We all know that Arya bhatta is the greatest mathematics belongs to india . When his daughter

Mayabati was in her teen age he discovered a problem. At that time the age of mayabati is a prime number,let that age is a. After some years  her age becomes b. then Arya Bhatta was able to solve that problem wit the help of he daughter mayabati. If a-b=5 & product of a& b is 26 then what is the sum of two squares?

A)77 b) 45 c)89 d)67

Page 16: Aptitude Questions

16

 6. How many 13 digit numbers are possible by using the digits 1,2,3,4,5 which are divisible by 4 if

repetition of digits is allowed? 5 to the power 12 7. (40*40* 40-31*31*31)/(40*40+40*31+31*31)=?a simle calcutation 8. x/2y=2a,then 2x/x-2ay=?(some thing like this .very easy ) 11. 1st a story. Then a simple ratio problem. The question was if the ratio of age of two persons is

5:6,sum of present age is 33,then in how many years the ratio of their age becomes 7:8?a)3 b)4 c)5 d)6 14. Simple question between big one on average age. 5th like a,b,c weigheted separately

1st a,b,c ,then a& b,then b&c ,then c&a at last abc, the last weight was 167, then what will be the avg weight of the 7 weight?

  Observe the dilemma of the fungus; it is a plant, but possesses no chlorophyll. While all other plants

put the sun’s energy to work for them combining the nutrients of ground and air into the body structure._____1______.Chlorophyll is found in these other plants which, having received their energy free from the sun, relinquish it__2__.

 In this search of energy, the fungus has become the earth’s major source of rot and decay. Wherever

you see mould forming on   a piece of bread, or a pile of leaves turning to compost, or a bloom down tree becoming pulp on the ground, _____3___.

With fungus action, the earth would be pooled high with_____4______. In fact, certain plants which contain resins that are ____5_______. Specimen of the redwood tree, for

instance can still be found ______6________.A. You are watching a fungus actingB. Resting on the forest floor centuries after having been cut downC. Responsible for decomposition of much plant lifeD. Fungus must look elsewhere for an energy supplyE. Cannot produce their own store of energyF. The dead plant life of past centuriesG. At some point in their cycle either to animal or to fungiH. Fungus is vastly different from other plantsI. Toxic to fungi will last indefinitely.One large passage-6marks.

I answer all question from passage in  guess becos i have no time to read whole passage care fully.

1. Two pencils costs 8 cents, then 5 pencils cost how muchSol: 2 pencils è 8 cents => 1 pencil è 4 centsTherefore 5 pencils cost = 5 * 4 = 20 cents

2. A work is done by two people in 24 min. one of them can do this work a lonely  in 40 min. how much time required to do the same work for the second person.Sol: (A+B) can do the work in = 1/24 min.A alone can do the same work in = 1/40 min.B alone can do the same work in = (A+B)’s – A’s = 1/24 – 1/40 = 1/60Therefore, b can do the same work in = 60 min

3.A car is filled with four and half gallons of oil for full round trip. Fuel is taken  1/4 gallon more in going than coming. What is the fuel consumed in coming up?Sol Before the trip, car is filled with = 4 ½ gallon of oilLet ‘X’ be the quantity of fuel consumed for the trip in one directionThe fuel consumed while going = X + ¼The fuel consumed while coming = XTherefore, the fuel consumed for the trip = (X + ¼) + X = 4 ½

Page 17: Aptitude Questions

17

ð 2X + ¼ = 4 ½ => 2X = 4 ½ - ¼ => 2X = 4 ¼ => X= 2. approxTherefore the fuel consumed while coming = 2 gallon

4. Low temperature at the night in a city is 1/3 more than 1/2 high as higher  temperature in a day. Sum of the low temperature and highest temp. is 100 degrees. Then what is the low temp?Sol: 40 deg.

5. A person, who decided to go to weekend trip should not exceed 8 hours driving in a day. Average speed of forward journey is 40 m/h. Due to traffic in Sundays, the return journey average speed is 30 m/h. How far he can select a picnic spot?a) 120 milesb) Between 120 and 140 milesc) 160 milesAnswer: 120 miles

6. A person was fined for exceeding the speed limit by 10mph. Another person was also fined for exceeding the same speed limit by twice the same. If the second person was traveling at a speed of 35 mph, find the speed limit.Sol: Let ‘x’ be the speed limitPerson ‘A’ was fined for exceeding the speed limit by = 10mphPerson ‘B’ was fined for exceeding the speed limit by = twice of ‘A’= 2*10mph=20mph given that the second person was traveling at the speed of 35mph => 35mph – 20mph = 15mphTherefore the speed limit is =15 mph

7.A bus started from bus stand at 8.00am, and after 30 minutes staying at destination, it returned back to the bus stand. The destination is 27 miles from the bus stand. The speed of the bus is 18mph. In return journey bus travels with 50% fast speed. At what time it returns to the bus stand?Sol: 11.00am

8.In a mixture, R is 2 parts S is 1 part. In order to make S to 25% of the mixture, how much r is to be added?Sol: One Part

9. Wind flows 160 miles in 330 min, for 80 miles how much time required.Sol:

10. With 4/5 full tank vehicle travels 12 miles, with 1/3 full tank how much distance travelsSol: ( 5 miles )

11. A storm will move with a velocity of towards the center in hours, at the same rate how much far will it move in hrs.Sol: ( but the answer is 8/3 or 2 2/3 )

12. In a two-dimensional array, X (9, 7), with each element occupying 4 bytes of memory, with the address of the first element X (1, 1) is 3000; find the address of X (8, 5).Ans: 3212

13. In the word ORGANISATIONAL, if the first and second, third and forth, forth and fifth, fifth and sixth words are interchanged up to the last letter, what would be the tenth letter from right?Ans:I

14. What is the largest prime number that can be stored in an 8-bit memory? Ans : 25115. Select the odd one out…..a. Java b. Lisp c. Smalltalk d. Eiffel.16. Select the odd one out a. SMTP b. WAP c. SAP d. ARP17. Select the odd one out a. Oracle b. Linux c. Ingress d. DB218. Select the odd one out a. WAP b. HTTP c. BAAN d. ARP19. Select the odd one out a. LINUX b. UNIX c. SOLARIS d. SQL SEVER20. Select the odd one out a. SQL b. DB2 c. SYBASE d. HTTP21. The size of a program is N. And the memory occupied by the program is given by M = square root

of 100N. If the size of the program is increased by 1% then how much memory now occupied?Ans: 0.5%(SQRT 101N)

22. A man, a woman, and a child can do a piece of work in 6 days. Man only can do it in 24 days. Woman can do it in 16 days and in how many days child can do the same work?Ans: 16

23. In which of the system, decimal number 184 is equal to 1234?Ans: 5

24. Find the value of the 678 to the base-7.Ans: 1656

Page 18: Aptitude Questions

18

25. Number of faces, vertices and edges of a cube Ans: 6 8 12

26. Complete the series 2, 7, 24, 77,__Ans: 238

27. Find the value of @@+25-++@16, where @ denotes "square" and + denotes "square root".Ans: 621

28. Find the result of the following expression if, M denotes modulus operation, R denotes round-off, T denotes truncation: M(373,5)+R(3.4)+T(7.7)+R(5.8)Ans:19

29. If TAFJHH is coded as RBEKGI then RBDJK can be coded as?Ans: qcckj

30. G(0)= -1, G(1)=1, G(N)=G(N-1) - G(N-2), G(5)= ?Ans: - 2

31. What is the max possible 3 digit prime number?Ans: 997

32. A power unit is there by the bank of the river of 750 meters width. A cable is made from power unit to power plant opposite to that of the river and 1500mts away from the power unit.The cost of the cable below water is Rs.15/- per meter and cost of cable on the bank is Rs.12/-per meter. Find the total of laying the cable.Ans : 1000 (24725-cost)

33. The size of a program is N. And the memory occupied by the program is given by M = square root of 100N. If the size of the program is increased by 1% then how much memory now occupied?Ans:0.5%(SQRT 101N)

34. In Madras , temperature at noon varies according to -t^2/2 + 8t + 3, where t is elapsed time. Find how much temperature more or less in 4pm to 9pm.Ans: At 9pm 7.5 more

35. The size of the bucket is N kb. The bucket fills at the rate of 0.1 kb per millisecond. A programmer sends a program to receiver. There it waits for 10 milliseconds. And response will be back to programmer in 20 milliseconds. How much time the program takes to get a response back to the programmer, after it is sent?Ans: 30

36. A man, a woman, and a child can do a piece of work in 6 days. Man only can do it in 24 days. Woman can do it in 16 days and in how many days child can do the same work?Ans: 16

37. Which of the following are orthogonal pairs?a. 3i+2j b. i+j c. 2i-3j d. -7i+jAns: a, c

38. If VXUPLVH is written as SURMISE, what is SHDVD?Ans: PEASE

39. If A, B and C are the mechanisms used separately to reduce the wastage of fuel by 30%, 20% and 10%.What will be the fuel economy if they were used combined.Ans: 20%

40. What is the power of 2? a. 2068 b.2048 c.266841. Complete the series. 3, 8, --, 24, --, 48, 63. Ans: 15.3542. Complete the series. 4, -5, 11, -14, 22, -- Ans: -2743. A, B and C are 8 bit no's. They are as follows:

A -> 1 1 0 1 1 0 1 1B -> 0 1 1 1 1 0 1 0C -> 0 1 1 0 1 1 0 1Find ((A-B) u C)=? Hint: 109…. A-B is {A} - {A n B}

44. A Flight takes off at 2 A.M from northeast direction and travels for 11 hours to reach the destination, which is in northwest direction. Given the latitude and longitude of source and destination. Find the local time of destination when the flight reaches there?Ans: 7 am

45. A can copy 50 papers in 10 hours while both A & B can copy 70 papers in 10 hours. Then for how many hours required for B to copy 26 papers?Ans: 13

Page 19: Aptitude Questions

19

46. A is twice efficient than B. A and B can both work together to complete a work in 7 days. Then find in how many days, A alone can complete the work?Ans: 10.5

47. A finish the work in 10 days. B is 60% efficient than A. So how many days do B takes to finish the work?Ans :100/6

48. A finishes the work in 10 days & B in 8 days individually. If A works for only 6 days then how many days should B work to complete A's work?Ans: 3.2 days

49. Given the length of the 3 sides of a triangle. Find the one that is impossible? (HINT: sum of smaller 2 sides is greater than the other one, which is larger)

50. Find the singularity matrix from a given set of matrices? (Hint det(A)==0))51. (Momentum*Velocity)/(Acceleration * distance). Find units.

Ans: mass52. The number 362 in decimal system is given by (1362) x in the X System of numbers find the value

of X a} 5 b) 6 c) 7 d) 8 e) 953. Given $ means Tripling and % means change of sign then find the value of $%$6-%$%654. My flight takes of at 2am from a place at 18N 10E and landed 10 Hrs later at a place with

coordinates 36N70W. What is the local time when my plane landed?6:00 am b) 6:40am c) 7:40 d) 7:00 e) 8:00(Hint: Every 1 deg longitude is equal to 4 minutes. If west to east add time else subtract time)

55. 7 8 9 11 13 ?? 19 56. there is a question like log.42=…,log.43=….then log.41==?1)In a two-dimensional array, X (8, 7), with each element occupying 2 bytes of memory, with the

address of the first element X (1, 1) is 3000, find the address of X (2, 3). Ans : 32122)12) Which of the following are orthogonal pairs? a. 3i+5j b. i+j c. 5j-3i d. -7i+j Ans : (A) & (C).3) Which of the following set of numbers has the highest Standard deviation? a)3,0,3,0,3,0                     b) -3, -3, -3, -3, -3, -3 c)3,3,3,3,3,3                     d) -3,3,-3, 3,-3, 3Ans : D4) (momentum * velocity)/(force * time) = velocity5)33). In the word ORGANISATION, if the first and second, third and forth, fifth and sixth, etc words are interchanged, what would be the 8th letter from right? (Ans: ‘I’)6) If A, B, C are the mechanisms used separately to reduce the wastage of fuel by 30%, 40%, 10%.What will be the fuel economy if they were used combine? 1.68.4         2.62.2         3.58         4.27 (Ans- 62.2)          sol:  (70/100)*(60/100)*(90/100)*100=37.8 Eco = (100-37.8) =62.27) What is the value of the following expression M(973,5)+T(7.7)+R(4.4)-T(3.6) Where M- MODULAS R- ROUNDOFF T- TRUNCATE (Ans : 11) Sol: 3 + 7+ 4 - 3 ==11 8) FIND THE MISSING NO. IN  SERIES ?3 8 15   ? 61 124 Ans: 309) If G(0) = 1 and G(N)=N*G(N-1) then What is the value  of G(4)?(Ans: 24)10)  Match the following (this type of question but not same) A B 1. Mammal, cow                    --->                 a. A type of2. Snake reptile                      --->                 b. A part of3. Roof - Building                   --->                 c. Not a type of4. Mushroom - Vegetables     --->                 d. A superset of (Ans: 1-d, 2-c, 3-b, 4-a) 11)In Chennai , temperature at noon varies according to -t^2/2 + 8t + 3, where t is elapsed time. Find how much temperature more or less in 4pm to 9pm.Ans. (put 9 & 4in eq. n subtract ) 12). Find the value of @@+25 - ++@16, where @ denotes "square" and + denotes "square root".

Page 20: Aptitude Questions

20

Ans. 62113) Find the fourth row, having the bit pattern as an integer in an 8-bit computer, and express the answer in its decimal value.A 0 0 0 0 1 1 1 1B 0 0 1 1 0 0 1 1C 0 1 0 1 0 1 0 1(A AND B )OR C)) ?Ans: 4714) An aircraft takes off from A (89o N Lat, 10o E ) at 6.00 AM local time to B (50o S , 70o W ). If the flying time is 10 hours what is the local time of landing at B?Ans:10:40am(if plane is going frm west to east then then Add those hrs)

15). Select the odd one out a. SMTP b.WAP c. SAP d.ARPAns : c16). select odd one- sql,db2,sybase,http {ans-htttp} 17). select odd one-sybase,db2,oracle,unix {ans-unix} 18-20)THREE QUESTION FROM VENN DIAGRAM(like 30 speaks eng,42 speaks french,.......)21-23)THREE QUESTION FROM PIE CHART......24)What is the largest prime number that can be stored in an 9-bit memory?Ans: 50925)In a triangle which one is not possible. A(30*,60*,90*), B(47*,69*,64* ), C(30*,50*,110* ), D(60*,80*,40*)?Ans: C26)11) A power unit is there by the bank of the river of 900 meters width. A cable is made from power unit to power a plant opposite to that of the river and 3000mts away from the power unit. The cost of the cable below water is Rs. 5/-per meter and cost of cable on the bank is Rs.3/- per meter. Find the the cable from which it will cut.Ans: 27)10) In which of the system, decimal number 194 is equal to 1234? Ans: 5so on.............................4. Simple puzzle based on IQ 3 persons a,b,c were there A always says truth, B lies on Monday, Tuesday,& Wednesday. But C lies on Thrusday, Friday & Saturday. One day A said ”that B & C said to A that” B said “yesterday way one of the days when I lies”,C said that ”yesterday way one of the days when I lies too”. Then which day was that?Ans: a Sunday 

 1) If log 0.317=0.3332 and log 0.318=0.3364 then find log 0.319 = Sol: Given log 0.317=0.3332 and log 0.318=0.3364 Then Log 0.319=log0.318+ (log0.318-log0.317) =0.3396   2) A box of 150 packets consists of 1kg packets and 2kg packets. Total weight of box is 264kg. How many 2kg packets are there? Sol: Given x= 2 kg Packs y= 1 kg packs  

Page 21: Aptitude Questions

21

 => x + y = 150 .......... Eqn 1 => 2x + y = 264 .......... Eqn 2 On solving these two equations x = 114 By using equation 1 114 + y = 150 => y = 36   =>Number of 2 kg Packs = 114.   3) My flight takes of at 2am from a place at 18N 10E and landed 10 Hrs later at a place with coordinates 36N70W. What is the local time when my plane landed? a) 6:00 am b) 6:40am c) 7:40 d) 7:00 e) 8:00 Sol: (Hint: Every 1 deg longitude is equal to 4 minutes. If west to east add time else subtract time) Ans: 8:00   4) A Flight takes off at 2 A.M from northeast direction and travels for 11 hours to reach the destination, which is in northwest direction. Given the latitude and longitude of source and destination. Find the local time of destination when the flight reaches there? Ans: 7 AM (or) 1 PM 5) A moves 3 kms east from his starting point. He then travels 5 kms north. From that point he moves 8 kms to the east. How far is A from his starting point? Ans: 13 kms 6) Aeroplane is flying at a particular angle and latitude, after some time latitude is given. (8 hrs later), u r asked to find the local time of the place. 7) An Aeroplane starts from A (SOME LATITUDE IS GIVEN ACCORDING TO PLACE).At 2 AM local time to B (SOME LATITUDE). Traveling time is 10 Hours. What is the local time of B when it reaches B? 8) A plane moves from 9°N40°E to 9°N40°W. If the plane starts at 10 am and takes 8 hours to reach the destination, find the local arrival time. Sol: Since it is moving from east to west longitude we need to add both Ie, 40+40=80 Multiply the ans by 4 

Page 22: Aptitude Questions

22

=>80*4=320min Convert this min to hours i.e., 5hrs 33min It takes 8hrs totally. So 8-5hr 30 min=2hr 30min So the ans is 10am+2hr 30 min Ans: 12:30 it will reach  9) The size of the bucket is N kb. The bucket fills at the rate of 0.1 kb per millisecond. A programmer sends a program to receiver. There it waits for 10 milliseconds. And response will be back to programmer in 20 milliseconds. How much time the program takes to get a response back to the programmer, after it is sent? Sol: The time being taken to fill the bucket. After reaching program it waits there for 10ms and back to the programmer in 20 ms. then total time to get the response is 20ms +10 ms=30ms Ans: 30ms  10) A file is transferred from one location to another in ,buckets,. The size of the bucket is 10 kilobytes. Eh bucket gets filled at the rate of 0.0001 kilobytes per millisecond. The transmission time from sender to receiver is 10 milliseconds per bucket. After the receipt of the bucket the receiver sends an acknowledgement that reaches sender in 100 milliseconds. Assuming no error during transmission, write a formula to calculate the time taken in seconds to successfully complete the transfer of a file of size N kilobytes. Ans: (n/1000)*(n/10)*10+ (n/100).... (Not 100% sure)  11)A fisherman,s day is rated as good if he catches 9 fishes ,fair if 7 fishes and bad if 5 fishes .He catches 53 fishes in a week n had all good, fair n bad days in the week. So how many good, fair n bad days did the fisher man had in the week. Sol:good days means --- 9 fishes so 53/9=4(remainder=17) if u assume 5 then there is no chance for bad days. fair days means ----- 7 fishes so remaining 17 --- 17/7=1(remainder=10) if u assume 2 then there is no chance for bad days. bad days means -------5 fishes so remaining 10---10/5=2days. 4*9=36 7*1=7 2*5=10 36+7+10=53... Ans: 4 good, 1 fair, 2bad. ==== total 7 days.  12) x+y+z=7--------- eq1 

Page 23: Aptitude Questions

23

9*x+7*y+5*z=53 -------eq2 Sol: Multiply eq 1 by 9, 9*x+9*y+9*z=35 -------------eq3 From eq2 and eq3 2*y+4*z=10-----eq4 Since all x, y and z are integer i should put a integer value of y such that z sud be integer in eq 4.....And there will be two value y=1 or 3 then z = 2 or 1 from eq 4 For first y=1,z=2 then from eq1 x= 4 So 9*4+1*7+2*5=53.... Satisfied Now for second y=3 z=1 then from eq1 x=3 So 9*3+3*7+1*5=53 ......satisfied So finally there are two solution of this questioAns:(x,y,z)=(4,1,2) and (3,3,1)...  13) Y catches 5 times more fishes than X. If total number of fishes caught by X and Y is 42, then number of fishes caught by X? Sol: let no. of fish x catches=p No. caught by y =r r=5p. Given r+p=42 Then p=7, r=35 14) Three companies are working independently and receiving the savings 20%, 30%, 40%. If the companies work combine, what will be their net savings? Sol: Suppose total income is 100 So amount x is getting is 80y is 70 z =60 Total=210But total money is 300 300-210=90 So they are getting 90 rs less 90 is 30% of 300 so they r getting 30% discount   

Page 24: Aptitude Questions

24

15) The ratio of incomes of C and D is 3:4.the ratio of their expenditures is 4:5.Find the ratio of their savings if the savings of C is one fourths of his income? Sol: incomes: 3:4 Expenditures: 4:5 3x-4y=1/4(3x) 12x-16y=3x 9x=16y y=9x/16 (3x-4(9x/16))/ ((4x-5(9x/16))) Ans: 12/19   16)If A can copy 50 pages in 10 hours and A and B together can copy 70 pages in 10 hours, how much time does B takes to copy 26 pages? Sol: A can copy 50 pages in 10 hrs. =>A can copy 5 pages in 1hr. (50/10) Now A & B can copy 70 pages in 10hrs. Thus, B can copy 90 pages in 10 hrs. [Eqn. is (50+x)/2=70, where x--> no. of pages B can copy in 10 hrs.] So, B can copy 9 pages in 1hr. Therefore, to copy 26 pages B will need almost 3hrs. Since in 3hrs B can copy 27 pages17) A can copy 50 papers in 10 hours while both A & B can copy 70 papers in 10 hours. Then for how many hours required for B to copy 26 papers? ANS: 13 18) A is twice efficient than B. A and B can both work together to complete a work in 7 days. Then find in how many days A alone can complete the work? ANS: 10.5 (11)19) A finish the work in 10 days. B is 60% efficient than A. So how days does B take to finish the work? Ans: 100/6 (4 days)   20) A finishes the work in 10 days & B in 8 days individually. If A works for only 6 days then how many days should B work to complete A,s work? Ans: 3.2 days (4 days)

Page 25: Aptitude Questions

25

21) A man, a woman, and a child can do a piece of work in 6 days. Man only can do it in 24 days. Woman can do it in 16 days and in how many days child can do the same work? Ans: 16 22) If 20 men take 15 days to complete a job, in how many days can 25 men finish that work? Ans. 12 days23) One fast typist type some matter in 2hr and another slow typist type the same matter in 3hr. if both do combine in how much time they will finish. Ans: 1hr 12min 24) A man shapes 3 cardboards in 50 minutes, how many types of cardboard does he shape in 5 hours? Ans: 18cardboards25) A work is done by two people in 24 min. one of them can do this work a lonely in 40 min. how much time required to do the same work for the second person. Sol: (A+B) can do the work in = 1/24 min. A alone can do the same work in = 1/40 min. B alone can do the same work in = (A+B),s - A,s = 1/24 - 1/40 = 1/60 => B can do the same work in = 60 min Ans: 60 min 26) A can do a piece of work in 20 days, which B can do in 12 days. In 9 days B does ¾ of the work. How many days will A take to finish the remaining work?   27) Anand finishes a work in 7 days; Bittu finishes the same job in 8 days and Chandu in 6 days. They take turns to finish the work. Anand on the first day, Bittu on the second and Chandu on the third day and then Anand again and so on. On which day will the work get over? A) 3rd b) 6th c) 9th d) 7th 28) 3 men finish painting a wall in 8 days. Four boys do the same job in 7 days. In how many days will 2 men and 2 boys working together paint two such walls of the same size? A) 6 6/13 days B) 3 3/13 days C) 9 2/5 days D) 12 12/13 days  29) what,s the answer for that? A, B and C are 8 bit no,s. They are as follows: A -> 1 1 0 0 0 1 0 1 

Page 26: Aptitude Questions

26

B -> 0 0 1 1 0 0 1 1 C -> 0 0 1 1 1 0 1 0 (- =minus, u=union) Find ((A - C) u B) =? Sol: We have to find (A-C) U B To find A-C, We will find 2,s compliment of C and them add it with A, That will give us (A-C) 2,s compliment of C=1,s compliment of C+1 =11000101+1=11000110 A-C=11000101+11000110 =10001001 Now (A-C) U B is .OR. Logic operation on (A-C) and B 10001001 .OR. 00110011 The answer is = 10111011, Whose decimal equivalent is 187. 30) A = 10010001 B = 01101010 C = 10010110 (AuB)nC =? [(A union B) intersection C =?] 31) A =0 0 0 0 1 1 1 1 B =0 0 1 1 0 0 1 1 C =0 1 0 1 0 1 0 1 ( A U B ) n C Find the fourth row, having the bit pattern as an integer in an 8-bit computer, and express the answer in its decimal value. Ans: 29 32) A, B and C are 8 bit nos. They are as follows: A 1 1 0 1 1 0 1 1 B 0 1 1 1 1 0 1 0 C 0 1 1 0 1 1 0 1 Find ( (A-B) u C )=?

Page 27: Aptitude Questions

27

 Hint: 109 A-B is {A} - {A n B} Ans: 0 1 1 1 1 1 1 1 (DB) 33) If A, B and C are the mechanisms used separately to reduce the wastage of fuel by 30%, 20% and 10%. What will be the fuel economy if they were used combined. Ans: 20% 34) In the class of 40 students, 30 speak Hindi and 20 speak English. What is the lowest possible number of students who speak both the languages? (a) 5 (b) 20 (c) 15 (d) 10 (e) 30 35) In a two-dimensional array, X (9, 7), with each element occupying 4 bytes of memory, with the address of the first element X (1, 1) is 3000, find the address of X (8, 5). Sol: [HINT~ Formula=Base Add + Byte reqd {N (i-1) + (j-1)} Where, Base Add=3000; Byte reqd=4; N=no of columns in array=7; i=8; j=5; IN ROW MAJOR ORDER] Ans: 3212 36) If the vertex (5, 7) is placed in the memory. First vertex (1, 1),s address is 1245 and then address of (5, 7) is ---------- Ans: 1279 37) A 2D array is declared as A [9, 7] and each element requires 2 byte. If A [1, 1] is stored in 3000. Find the memory of A [8, 5]? Ans: 3106 38) One circular array is given (means the memory allocation takes place like a circular fashion) dimension (9X7) .starting address is 3000.find the address of (2, 3) Ans: 555 39) The size of a program is N. And the memory occupied by the program is given by M = square root of 100N. If the size of the program is increased by 1% then how much memory now occupied? Sol: N is increased by 1% Therefore new value of N=N + (N/100)

Page 28: Aptitude Questions

28

 =101N/100 M=sqrt (100 * (101N/100)) Hence, we get M=sqrt (101 * N) Ans: 0. 5 %( =SQRT 101N) 40) A bus started from bus stand at 8.00a m and after 30 min staying at destination, it returned back to the bus stand. The destination is 27 miles from the bus stand. The speed of the bus 50 percent fast speed. At what time it retur4ns to the bus stand. Sol: (this is the step by step solution :) A bus cover 27 mile with 18 mph in =27/18= 1 hour 30 min. And it wait at stand =30 min. After this speed of return increase by 50% so 50%of 18 mph=9mph Total speed of returning=18+9=27 Then in return it take 27/27=1 hour Then total time in journey=1+1:30+00:30 =3 hour So it will come at 8+3 hour=11 a.m. So Ans==11 a.m 41) A Flight takes off at 2 A.M from northeast direction and travels for 11 hours to reach the destination which is in North West direction. Given the latitude and longitude of source and destination. Find the local time of destination when the flight reaches there? Ans: 7 AM or 1.00 PM 42) My flight takes of at 2am from a place at 18N 10E and landed 10 Hrs later at a place with coordinates 36N70W. What is the local time when my plane landed? a) 6:00 am b) 6:40am c) 7:40 d) 7:00 e) 8:00 (Hint: Every 1 deg longitude is equal to 4 minutes. If west to east add time else subtract time) Ans: 8:00 43) A moves 3 kms east from his starting point. He then travels 5 kms north. From that point he moves 8 kms to the east. How far is A from his starting point? Ans: 13 kms

44) A plane moves from 9°N40°E to 9°N40°W. If the plane starts at 10 am and takes 8 hours to reach the destination, find the local arrival time. 

Page 29: Aptitude Questions

29

45) In Madras, temperature at noon varies according to -t^2/2 + 8t + 3, where t is elapsed time. Find how much temperature more or less in 4pm to 9pm. (May be we can solve it by Definite Integration. Check any way} Ans: at 9 pm 7.5 more or 385.8 (DB)46) For Temperature a function is given according to time: ((t**2)/6) + 4t +12 what is the temperature rise or fall between 4.AM TO 9 AM Sol: In equation first put t=9, We will get 34.5........................... (1) Now put t=4, We will get 27.............................. (2) So Ans=34.5-27 =7.5 47) For Temperature a function is given according to time: ((t**2)/6) + 4t +12 what is the temperature rise or fall between 5 PM to 8 PM 48) Low temperature at the night in a city is 1/3 more than 1/2 high as higher temperature in a day. Sum of the low tem. And highest temp is 100 degrees. Then what is the low temp? Sol: Let highest temp be x So low temp=1/3 of x of 1/2 of x plus x/2 i.e. x/6+x/2 Total temp=x+x/6+x/2=100 Therefore, x=60 Lowest temp is 40 Ans :( 40 deg.) 49) A person had to multiply two numbers. Instead of multiplying by 35, he multiplied by 53and the product went up by 540. What was the raised product? a) 780 b) 1040 c) 1590 d) 1720Sol: x*53-x*35=540=> x=30 therefore, 53*30=1590Ans: 1590 50) How many positive integer solutions does the equation 2x+3y = 100 have?  Sol: Given 2x+3y=100, take l.c.m of ,x, coeff and ,y, coeff i.e. l.c.m of 2,3 ==6then divide 100 with 6 , which turns out 16 hence answer is 16short cut formula--- constant / (l.cm of x coeff and y coeff)   51) The total expense of a boarding house is partly fixed and partly variable with the number of boarders. The charge is Rs.70 per head when there are 25 boarders and Rs.60 when there are 50 boarders. Find the charge per head when there are 100 boarders. 

Page 30: Aptitude Questions

30

Sol: let a = fixed cost and k = variable cost and n = number of boarders Total cost when 25 boarders c = 25*70 = 1750 i.e. 1750 = a + 25k Total cost when 50 boarders c = 50*60 = 3000 i.e. 3000 = a + 50k Solving above 2 eqns, 3000-1750 = 25k i.e. 1250 = 25k i.e. k = 50 Therefore, substituting this value of k in either of above 2 eqns we get a = 500 (a = 3000-50*50 = 500 or a = 1750 - 25*50 = 500) So total cost when 100 boarders = c = a + 100k = 500 + 100*50 = 5500 So cost per head = 5500/100 = 55   52) Amal bought 5 pens, 7 pencils and 4 erasers. Rajan bought 6 pens, 8 erasers and 14 pencils for an amount which was half more than what Amal had paid. What % of the total amount paid by Amal was paid for pens?  Sol: Let, 5 pens + 7 pencils + 4 erasers = x rupees So 10 pens + 14 pencils + 8 erasers = 2*x rupees Also mentioned, 6 pens + 14 pencils + 8 erasers = 1.5*x rupees So (10-6) = 4 pens = (2-1.5) x rupees So 4 pens = 0.5x rupees => 8 pens = x rupees So 5 pens = 5x/8 rupees = 5/8 of total (note x rupees is total amt paid by Amal) i.e. 5/8 = 500/8% = 62.5% Ans: 62.5%   53) I lost Rs.68 in two races. My second race loss is Rs.6 more than the first race. My friend lost Rs.4 more than me in the second race. What is the amount lost by my friend in the second race? Sol: x + x+6 = rs 68 2x + 6 = 68 2x = 68-6 2x = 62 x=31 x is the amt lost in I race

Page 31: Aptitude Questions

31

 x+ 6 = 31+6=37 is lost in second race Then my friend lost 37 + 4 = 41 Rs Ans: 41 Rs   54) A face of the clock is divided into three parts. First part hours total is equal to the sum of the second and third part. What is the total of hours in the bigger part? Sol: The clock normally has 12 hr Three parts x, y, z x+y+z=12 x=y+z 2x=12 x=6 So the largest part is 6 hrs Ans: 6 hrs 55) (1- 1/6) (1-1/7).... (1- (1/ (n+4))) (1-(1/ (n+5))) = ? Sol: Leaving the first numerator and last denominator, all the numerator and denominator will cancelled out one another. Ans: 5/ (n+5)  56) Ten boxes are there. Each ball weighs 100 gms. One ball is weighing 90 gms. i) If there are 3 balls (n=3) in each box, how many times will it take to find 90 gms ball? ii) Same question with n=10 iii) Same question with n=9 Sol: The chances are When n=3 (i) nC1= 3C1 =3 for 10 boxes.. 10*3=30 (ii) nC1=10C1=10 for 10 boxes ....10*10=100 (iii) nC1=9C1=9 for 10 boxes.....10*9=90  57) With 4/5 full tank vehicle travels 12 miles, with 1/3 full tank how much distance travels? Sol: 4/5 full tank= 12 mile 1 full tank= 12/ (4/5)

Page 32: Aptitude Questions

32

 1/3 full tank= 12/ (4/5)*(1/3) = 5 miles Ans: 5 miles  58) Wind flows 160 miles in 330min.for 80 miles how much time required 160 miles? Sol: 1 mile = 330/160 80 miles= (330*80)/160=165 min. Ans: 165 min. 59) A person was fined for exceeding the speed limit by 10mph.another person was also fined for exceeding the same speed limit by twice the same if the second person was traveling at a speed of 35 mph. find the speed limit Sol :( x+10) =(x+35)/2 Solving the eqn we get x=15 Ans: 15 60) A sales person multiplied a number and get the answer is 3 instead of that number divided by 3.what is the answer he actually has to get. Sol: Assume 1 1* 3 = 3 1*1/3=1/3 So he has to got 1/3 Ans: 1/3 61) The size of the bucket is N kb. The bucket fills at the rate of 0.1 kb per millisecond. A programmer sends a program to receiver. There it waits for 10 milliseconds. And response will be back to programmer in 20 milliseconds. How much time the program takes to get a response back to the programmer, after it is sent? Ans: 30 milliseconds2) A person who decided to go weekend trip should not exceed 8 hours driving in a day average speed of forward journey is 40 mph due to traffic in Sundays the return journey average speed is 30 mph. How far he can select a picnic spot. Ans: Between 120 and 140 miles 63) Car is filled with four and half gallons of oil for full round trip. Fuel is taken 1/4 gallons more in going than coming. What is the fuel consumed in coming up. Sol: Let fuel consumed in coming up is x. Thus equation is: x+1.25x=4.5Ans: 2gallons 

Page 33: Aptitude Questions

33

 64) 40% employees are male if 60% of supervisors are male so for 100% is 26.4%so the probability is ......Ans: 0.264 65) Gavaskar average in first 50 innings was 50. After the 51st innings his average was 51 how many runs he made in the 51st innings Sol: first 50 ings.- run= 50*50=2500 51st ings. - Avg 51. So total run =51*51=2601. So run scored in that ings=2601-2500=101 runs. Ans: 101 runs 66) Hansie made the following amounts in seven games of cricket in India: Rs.10, Rs.15, Rs.21, Rs.12, Rs.18, Rs.19 and Rs.17 (all figures in crores of course).Find his average earnings.Ans: Rs.16 crore 67) Average of 5 numbers is -10 sum of 3 numbers is 16, what is the average of other two numbers?Ans: -33 68) If A, B and C are the mechanisms used separately to reduce the wastage of fuel by 30%, 20% and 10%. What will be the fuel economy if they were used combined. Ans: 20% 69) In 80 coins one coin is counterfeit what is minimum number of weighing to find out counterfeit coin Sol: the minimum number of weightings needed is just 5.as shown below (1) 80->30-30 (2) 15-15 (3) 7-7 (4) 3-3 (5) 1-1Ans: 5. 70) 2 oranges, 3 bananas and 4 apples cost Rs.15. 3 oranges, 2 bananas, and 1 apple costs Rs 10. What is the cost of 3 oranges, 3 bananas and 3 apples? Sol: 2x+3y+4z=15 3x+2y+z=10 Adding 5x+5y+5z=25 x+y+z=5 that is for 1 orange, 1 banana and 1 apple requires 5Rs. So for 3 orange, 3 bananas and 3 apples require 15Rs. i.e. 3x+3y+3z=15 Ans: 1571) In 8*8 chess   board  what is the total number of squares refers odele discovered that there are 204 squares on the board .We found that you would add the different squares 

Page 34: Aptitude Questions

34

= 1 + 4 + 9 + 16+ 25 + 36 + 49 + 64. =204Also in 3*3 tic tac toe board what is the total no of squares Ans: 14 i.e. 9+4(bigger ones) +1 (biggest one) If you get 100*100 board just use the formula the formula for the sum of the first n perfect squares isn x (n + 1) x (2n + 1)72) One fast typist type some matter in 2hr and another slow typist type the same matter in 3hr. If both do combine in how much time they will finish. Sol: Faster one can do 1/2 of work in one hour slower one can do 1/3 of work in one hour both they do (1/2+1/3=5/6) the work in one hour. So work will b finished in 6/5=1.2 hour i e 1 hour 12 min. Ans: 1 hour 12 min. 73)If Rs20/- is available to pay for typing a research report & typist A produces 42 pages and typist B produces 28 pages. How much should typist A receive? Sol: Find 42 % of 20 rs with respect to 70 (i.e. 28 + 42) ==> (42 * 20)/70 ==> 12 Rs Ans: 12 Rs74) In some game 139 members have participated every time one fellow will get bye what is the number of matches to choose the champion to be held?Ans: 138 matches (Explanation: since one player gets a bye in each round, he will reach the finals of the tournament without playing a match. Therefore 137 matches should be played to determine the second finalist from the remaining 138 players (excluding the 1st player) Therefore to determine the winner 138 matches should be played.) 75) ONE RECTANGULAR PLATE WITH LENGTH 8INCHES, BREADTH 11 INCHES AND 2 INCHES THICKNESS IS THERE.WHAT IS THE LENGTH OF THE CIRCULAR ROD WITH DIAMETER 8 INCHES AND EQUAL TO VOLUME OF RECTANGULAR PLATE? Sol: Vol. of rect. plate= 8*11*2=176 Area of rod= (22/7)*(8/2)*(8/2) = (352/7) Vol. of rod=area*length=vol. of plate So length of rod= vol of plate/area=176/ (352/7) =3.5Ans: 3.5 76) One tank will fill in 6 minutes at the rate of 3cu ft /min, length of tank is 4 ft and the width is 1/2 of length, what is the depth of the tank? Ans: 3 ft 7.5 inches 77) A power unit is there by the bank of the river of 750 meters width. A cable is made from power unit to power a plant opposite to that of the river and 1500mts away from the power unit. The cost of the cable below water is Rs. 15/- per meter and cost of cable on the bank is Rs.12/- per meter. Find the total of laying the cable. Ans: 1000 (24725 - cost) 20250 Ans: Rs. 22,500 (hint: the plant is on the other side of the plant i.e. it is not on the same side as the river) 78) The cost of one pencil, two pens and four erasers is Rs.22 while the cost of five pencils, four pens and two erasers is Rs.32.How much will three pencils, three pens and three erasers cost? 

Page 35: Aptitude Questions

35

Sol :( let x b pencil, y b pen and z b eraser... u get x+2y+4z=22 and 5x+4y+2z=32 add 6x+6y+6z=54 div by 2 you get 27) Ans: 27 79) A man has to get air-mail. He starts to go to airport on his motorbike. Plane comes early and the mail is sent by a horse-cart. The man meets the cart in the middle after half an hour. He takes the mail and returns back, by doing so, he saves twenty minutes. How early did the plane arrive? Sol: Assume he started at 1:00, so at 1:30 he met cart .He returned home at 2:00.so it took him 1 hour for the total journey. By doing this he saved 20 min. So the actual time if the plane is not late is 1 hour and 20 min. So the actual time of plane is at 1:40.The cart traveled a time of 10 min before it met him. So the plane is 10 min early. Ans: 10 min 80) Ram singh goes to his office in the city every day from his suburban house. His driver Mangaram drops him at the railway station in the morning and picks him up in the evening. Every evening Ram singh reaches the station at 5 o,clock. Mangaram also reaches at the same time. One day Ram singh started early from his office and came to the station at 4 o,clock. Not wanting to wait for the car he starts walking home. Mangaram starts at normal time, picks him up on the way and takes him back house, half an hour early. How much time did Ram singh walked? 81) 2 trees are there. One grows at 3/5 of the other. In 4 years total growth of the trees is 8 ft. what growth will smaller tree have in 2 years. Sol: THE BIG TREE GROWS 8FT IN 4 YEARS=>THE BIG TREE GROWS 4FT IN 2 YEARS.WHEN WE DIVIDE 4FT/5=.8*3=>2.44 (x+ (3/5) x) =88x/5=2x=5/4 After 2 years x= (3/5)*(5/4)*2 =1.5 (less than 2 feet) 82) There is a six digit code. Its first two digits, multiplied by 3 gives all ones. And the next two digits multiplied by 6 give all twos. Remaining two digits multiplied by 9 gives all threes. Then what is the code? Sol: Assume the digit xx xx xx (six digits) First Two digit xx * 3=111 xx =111/3=37 (First two digits of 1 is not divisible by 3 so we can use 111) Second Two digit xx*6=222 xx=222/6=37 (First two digits of 2 is not divisible by 6 so we can use 222) Third Two digit xx*9=333 xx=333/9=37 (First two digits of 3 is not divisible by 9 so we can use 333) 83) There are 4 balls and 4 boxes of colors yellow, pink, red and green. Red ball is in a box whose color is same as that of the ball in a yellow box. Red box has green ball. In which box you find the yellow ball? Sol: Yellow box can have either of pink/yellow balls. 

Page 36: Aptitude Questions

36

if we put a yellow ball in "yellow" box then it would imply that "yellow" is also the color of the box which has the red ball(because according 2 d question,d box of the red ball n the ball in the yellow box have same color) Thus this possibility is ruled out... Therefore the ball in yellow box must be pink, hence the color of box contain in red ball is also pink.... =>the box color left out is "green", which is allotted to the only box left, the one which has yellow ball. Ans: green 84) A bag contains 20 yellow balls, 10 green balls, 5 white balls, 8 black balls, and 1 red ball. How many minimum balls one should pick out so that to make sure the he gets at least 2 balls of same color. Sol: suppose he picks 5 balls of all different colors then when he picks up the sixth one, it must match any on of the previously drawn ball color. Thus he must pick 6 balls Ans: he should pick 6 balls totally. 85) WHAT IS THE NUMBER OF ZEROS AT THE END OF THE PRODUCT OF THE NUMBERS FROM 1 TO 100? Sol: For every 5 in unit place one zero is added so between 1 to 100 there are 10 nos like 5,15,25,..,95 which has 5 in unit place. Similarly for every no divisible by 10 one zero is added in the answer so between 1 to 100 , 11 zeros are added For 25, 50, 75 3 extra zeros are added So total no of zeros are 10+11+3=24 86) There are two numbers in the ratio 8:9. If the smaller of the two numbers is increased by 12 and the larger number is reduced by 19 thee the ratio of the two numbers is 5:9. Find the larger number? Sol: 8x: 9x initially 8x+ 12: 9x - 19 = 5x: 9x 8x+12 = 5x > x = 4 9x = 36 (NOT SURE ABOUT THE ANSWER) 87) There are three different boxes A, B and C. Difference between weights of A and B is 3 kgs. And between B and C is 5 kgs. Then what is the maximum sum of the differences of all possible combinations when two boxes are taken each time Sol: A-B = 3 B-c = 5A-c = 8 so sum of diff = 8+3+5 = 16 kgs 88) A and B are shooters and having their exam. A and B fall short of 10 and 2 shots respectively to the qualifying mark. If each of them fired at least one shot and even by adding their total score together, they fall short of the qualifying mark, what is the qualifying mark? Sol: Because each had at least 1 shot done so 10 + 1 = 11 And 9 + 2 = 11 So the ans is 11 89) A, B, C, and D tells the following times by looking at their watches. A tells it is 3 to 12. B tells it is 3 past 12. C tells it is 12:2. D tells it is half a dozen too soon to 12. No two watches show the same time. The difference between the watches is 2,3,4,5 respectively. Whose watch shows maximum time? Sol: A shows 11:57, B shows 12:03, C shows 12:02 and D shows 11:06 therefore,

Page 37: Aptitude Questions

37

 Max time is for B 90) Falling height is proportional to square of the time. One object falls 64cm in 2sec than in 6sec from how much height the object will fall. Sol: The falling height is proportional to the square of the time. Now, the falling height is 64cm at 2sec So, the proportional constant is=64/ (2*2) =16; So, at 6sec the object fall maximum (16*6*6) cm=576cm; Now, the object may be situated at any where. If it is>576 only that time the object falling 576cm within 6sec .Otherwise if it is situated  a. A type of 2. Square - Polygon ---> b. A part of 3. Roof - Building ---> c. Not a type of 4. Mushroom - Vegetables ---> d. A superset of Ans: 1- d, 2- a, 3- b, 4- c 118) Match the following. 1. Brother - sister ---> a. Part of 2. Alsatian - dog ---> b. Sibling 3. Sentence - paragraph ---> c. Type of 4. Car - steering ---> d. Not a type of Ans. 1-b, 2-c, 3-a, 4-d 119) Match the following 1) Scooter --------- Automobile A. A PART OF 2).Oxygen ---------- Water B. A Type of 3).Shop staff ---------- Fitters C. NOT A TYPE OF 4). Bug ---------- Reptile D. A SUPERSET OF Ans. 1-b, 2-a, 3-d, 4-c 120) What is the largest prime number stored in a- -----> 6 bit pattern (ANS~2^6=64, so no is 61) ------> 7 bit pattern (ANS~2^7=128, so no is 127) -------> 8 bit pattern (ANS~2^8=256, so no is 251) -------->9 bit pattern (ANS~2^9=512, so no is 503)

Page 38: Aptitude Questions

38

 121) What is the max 3 digit Prime no? ANS=997  122) G(0)= -1, G(1)=1, G(N)=G(N-1) - G(N-2), G(5)= ?Ans - 2123) G (0) =1 G (1) = -1 IF G (N) =2* (G (N-1)) - 3(G (N-2)) Then what is the value of G (4)? 124) If f (0) =1 and f (n) = f (n-1)*n, find the value of f (4). Ans: 24 125) If g (0) =g (1) =1 and g (n) = g (n-1) + g (n -2) find g (6); 126) What is the power of 2? a. 2068 b.2048 c.2668 Ans: 2048 127) 8 to the power of x is 32, what is the value of x? 128) Power of 4 Ans-4096 129) Which one will be the exact power of 3? (i) 2768 (ii) 2678 (iii) 2187 130) Complete the series- a )3,8,a,24,b,48,63 [ ANS~ a=15, b=35 ] [HINT~DIFFERENCE IS 5, 7, 9, 11, 13, 15] B )26,19,17,13,11, ,8,7 [ ANS=9] [HINT~26,17,11,8 DECREASING LIKE 9,6,3 & 19,13,9,7 DECREASING LIKE 6, 4, 2] c)9,10,11,13,15, ,21,28 [ ANS=19 ] [HINT~9, 11, 15, and 21 INCREASING LIKE 2, 4, 6 & 10,13,19,28 INCRESING LIKE 3, 6, and 9] D) 4, -5, 11, -14, 22, --- [ ANS= -27] 131) Number of faces, vertices and edges of a cube ANS: 6,8,12 132) Find the value of- a) @@+25-++@16, where @ denotes"square" & + denotes "square root". [ANS=621] b) $%$6-%$%6, where $ means "tripling" & % means "change of sign". [ANS= -72] c) % # % 6 + # %# 6, % means "doubling" & # mean "reciprocal".

Page 39: Aptitude Questions

39

 132) Select odd one out 1) LINUX, WINDOWS   98 , SOLARIS, SMTP (ANS: SMTP) 2)MVS 3).JAVA b) LISP c) Smaltalk d)Eiffle Ans: LISP ( All other languages are OOPS) 4)1.http 2.arp 3.snmp 4.sap Ans:sap 5)1.linux 2.windows NT 3.sql server 4.Unix Ans: Sql server 6)1.SAP 2.ARP 3.WAP 4.TCP IP 7)a. Oracle b. Linux c. Ingress d. DB2 a. SMTP b. WAP c. SAP d. ARP Ans:SAP 9)a. WAP b. HTTP c. BAAN d. ARP Ans:Baan 10)a. LINUX b. UNIX c. SOLARIS d. SQL SERVER Ans:SQL SERVER 11)a. SQL b. DB2 c. SYBASE d. HTTP Ans: HTTP 12)a. Oracle b. Linux c. Ingress d. DB2 ANS:LINUX 133) Find the singularity matrix from a given set of matrices? (Hint det (A) ====0) 134) Which of the following are orthogonal pairs? a. 3i+2j b. i+j c. 2i-3j d. -7i+j Ans: a, c 135) (a) 2+3i (b) 1+i (c) 3-2i (d) 1-7i .Find which of the above is orthogonal. Ans: a, c 136) Sum of slopes of 2 perpendicular st. lines is given. Find the pair of lines from the given set of options which satisfy the above condition? 137) If Rs.1260 is divided between A, B and C in the ratio 2:3:4, what is C,s share? Ans: Rs. 560 138)A sum of money is divided among A, B and C such that for each rupee A gets, B gets 65paise and C gets 35paise. If C,s share is Rs.560, the sum is . a) 2400 b) 2800 c) 3200 d) 3800139) Complete the series. 1) 3, 8, --, 24, --, 48, 63. Ans: 15, 35 2) Complete the series. 4, -5, 11, -14, 22, --- Ans - 27 3) SERIES: 2, 7, 24, 77, ------ (238) or (240) 4)77, 49, 36, 18,? Ans: 8 (7*7=49) (4*9=36) (3*6=1 (1*8=

Page 40: Aptitude Questions

40

 5) series: 5 6 7 8 10 11 14?? Ans.15 or 18 6)15 14 12 11?? 9 8 Ans.10 7) what is the 12th term of the series 2, 5, 8 ... Ans. 35 8)58, 27, 12, x, 2, 1. Find x. 9)7, 9,13,_,27,37. Ans-19 10)2, 5, __, 19, 37, 75 Ans: 9 11) Complete the sequence 9, 10,11,13,15, __, 21, 28. 140) UNITS 1) (Momentum*Velocity)/ (Acceleration * distance) find units. Ans: mass 2) (energy * time * time)/ (mass * dist) = distance 3) (momentum * velocity)/ (force * time) = velocity 4) Find the physical quantity in units from the equation: (Force*Distance)/ (Velocity*Velocity) Ans. Ns2/m 5) Find the physical quantity represented by MOMENTUM *VELOCITY] / [LENGTH * ACCELERATION]?141) Find the result of the following _expression if, M denotes modulus operation, R denotes round-off, T denotes truncation: M (373, 5) +R (3.4) +T (7.7) +R (5.8) ANS: 19 142) Which of the following highest Standard deviation a) 7, - 7, 7,-7, 7,-7 b) 7, 7, 7,7,7,7 c) -7, - 7, -7,-7,-7,-7 d) -7, 7, -7, 7,-7, 7 Ans: d 143) 232 expressed in base-5 is Ans: 1412 144) A building with height D shadow up to G. A neighbor building with what height shadows C feet. |----|----|----|----|----|----|----| A B C D E F G H Sol: B Ft. or CD/G145) In a fraction, if 1 is added to both the numerator at the denominator, the fraction becomes 1/2. If numerator is subtracted from the denominator, the fraction becomes 3/4. Find the fraction. Ans. 3/7 146) The sum of the digits of a two digit number is 8. When 18 is added to the number, the digits are reversed. Find the numberAns. 35

Page 41: Aptitude Questions

41

 147) What number should be added to or subtracted from each term of the ratio 17 : 24 so that it becomes equal to 1 : 2. Ans. 10 should be subtracted Q No. 1  GARRULOUS(a) Arid                  (b) Hasty             (c) Sociable            (d) Quaint            (e) Talkative Q No. 2  INTER (a) Bury                  (b) Amuse           (c) Relate               (d) Frequent        (e) Abandon Q No. 3  CHIMERICAL (a) Developing        (b) Brief              (c) Distant              (d) Economical    (e) FantasticQ No. 4  COPIOUS(A) plentiful      (B)cheating       (C)dishonorable      (D)adventurous   (E)inspiredQ No. 5  EQUANIMITY (a) Clamour            (b) Disparity        (c) Composure       (d) Propensity      (e) Indivisibility Q No. 6  GALLEON (a) Liquid measure (b) Ship                (c) Armada             (d) Company       (e) Printer’s proof Q No. 7  REGAL (a) Oppressive        (b) Royal             (c) Major               (d) Basic             (e) Entertain Q No. 8  MUSTY (a) Stale                 (b) Necessary      (c) Indifferent          (d) Nonchalant    (e) Vivid Q No. 9  INDIGNITY (a) Pomposity         (b) Bombast        (c) Obeisance         (d) Insult              (e) Message Q No. 10 CANDID (a) Vague               (b) Outspoken     (c) Experience        (d) Anxious         (e) Sallow For each of the words in capital letters, choose from among the answers, the closest word that has the opposite meaning: Q No. 11 REPUDIATE (a) Hesitant             (b) Neutrality       (c) Admit                (d) Polite             (e) Incongruity Q No. 12 CREDIT (a) Believe false    (b) Treat as equal    (c) Make more difficult (d) Underemphasize  (e) Irresolute Q No. 13 TENACIOUS (a) Fast-running      (b) Intention         (c) Obnoxious        (d) Holding fast    (e) Irresolute Q No. 14 PRECIPITATE (a) Intricate             (b) Devious         (c) Posthumous       (d) Dilatory          (e) Contradictory Q No. 15 QUIXOTIC (a) Slow                 (b) Abstemious    (c) Pragmatic          (d) Benevolent     (e) Grave Q No. 16 ADMONITION (a) Premonition       (b) Hallucination (c) Escape               (d) Commendation  (e) Trepidation Q No. 17 PROCLIVITY (a) Prodigality         (b) Avoidance     (c) Credence          (d) Calumny        (e) Inception Q No. 18 OMNISCIENCE                           (a) Power               (b) Extravagance  (c) Magnanimity   (d) Conscience  (e) Ignorance

Page 42: Aptitude Questions

42

 Q No. 19 PRECIPITIOUS (a) Cooperative      (b) Cautious        (c) Inaccurate         (d) Formal           (e) Simplistic Q No. 20 PROTRACT (a) Defy                  (b) Supplement    (c) Postpone           (d) Shorten          (e) Design There was also a passage with 6 questions and fill in the blanks with 6 questions in this section.

Quantitative Aptitude Section :Q1.  Fill in the missing number in the sequence 7  8    9   11 13 ?? 19 26(a) 24  (b) 21   (c) 17   (d) 15  (e) 18Q2. In the following series, how many Us are there such that each U is followed by an G next to it if the G is not followed by a S next to it.U    G    U    S    Q    M    U    G    S    M    G    U    G    G    U    Q    M    U   U    G    Q    U    G    S    G    U    A     M   G    U    G    M(a)2 (b)3 (c)5 (d)4 (e)6Q3. If FLPMXQHO is coded as COMPUTER how will GFVMOXB be coded?(a) PROCESS (b) CENTRAL (c) PLEASE (d) SURMISE (e) DISPLAYQ4. Suppose the first and second letters in the word CONSEQUENCES were interchanged, also the third and fourth letters, the fifth and sixth etc.  Print the letter that would then be the eighth letter counting from the right.(a) C   (b) E     (c) N     (d) Q    (e) SQ5. How would the decimal number 362 be represented in a base -6 number System?  (a) 1234              (b) 1342             (c) 1362               (d) 1544          (e) 1645Q6. What is the largest prime number that can be stored in a 7-bit word computer?(a) 123                      (b) 129            (c) 121               (d) 127           (e) 119Q7. If n = 15 x 28 x 26, which of the following is NOT an integer?  a. n / 15  b. n / 21  c. n / 64  d. n / 35  e. n / 78Q8. Which of the following is a power of 2? a. 2148  b. 2096  c. 2048  d. 2444  e. 2198 Q9.       Pick the odd one out: (a) ORACLE  (b) SYBASE  (c) SMTP (d) DB2   (e)       INGRESS

Q10.    Pick the odd one out: (a) UNIX   (b) WINDOWS NT (c) LINUX(d) MVS   (e)        C++ Q11. The three numbers in brackets represent the length of the sides of a triangle. Which of these does not represent a proper triangle?(a) (5m,3m,4m)  (b) (4m,5m,4m) (c)  (4m,6m,8m) (d)  (6m,2m,2m) (e)  (12m,6m,8m)Q12. The three numbers in brackets in each of the following options represents the  number of edges ,the number of vertices  and the number of faces respectively. Find out which of these represents a solid planar cube? (a)  (4,8,12) (b)  (12,8,6) (c)  (8,6,12) (d)  (8,8,8)   (e)  (12,6,8)Q13. Which set of data exhibits a higher Standard Deviation? (a) 3, -3, 3, -3, 3, -3    (b) 3, 0, -3, 3, 0, -3 (c) 3, 3, 3, 0, 3, 3  (d) 3, 3, 3, 3, 3, 3  (e) -3, -3, -3, -3, -3, -3 The three circles below represent the number of people speaking French, German and English. Answer the next three questions based on the diagram(There was a ven diagram. This is very easy section. I have tried a lot to give u the diagram below)                            *   *       *   *                       *           *           *    <---English      --->       *          *     *           *  German    *    45    *  14   *      49   *    

Page 43: Aptitude Questions

43

                   *          *   *  *  *           *                   *          *   12    *    *      *                    *       *  *       *      *    *                      *    *     *   *    13      *                        *    16   *            *                           *  *        *  *    *                           *                  *                            *       51       *     <----French                              *           *                                  *   *                      Q14. How many more (or less) people speak English than people who speak German? (a) 2 (b) 3 (c) 1 (d) 5 (e) 6Q15. What percentage of people speaking French can also speak German but not English? (a) 16 (b) 14 (c) 17.4 (d) 15 (e)13Q16. What percentage of people can totally speak all three languages?  (a) 4 (b) 5 (c) 6 (d) 7 (e) 8

 The figure on the left represents number of members in a Club and the figure on the right depicts profession wise distribution for 1998

 Members (in "00s)Here was a bar chart .This is also very easy section. Q17. Which year has registered the maximum percentage growth in members?(a) 1996 (b) 1998 (c) 1994 (d) 1686 (e) 1888Q18. What are the average members for 1995-1999?(a) 7700 (b) 8800 (c)6800 (d)4800 (e) 7800Q19. If 10% of members in 1998 left the Club, how many fresh members were made in 1999? (a)4500 (b) 4900 (c)4600 (d) 4400 (e) 5400Q20. A hypothetical physical quantity is defined as              (Energy X Time X Time)                              (Mass X Distance) In what fundamental units would this quantity be expressed? (a)Time (b) Distance (c) Velocity (d) Mass (e) NoneQ21. A, B and C together can finish a piece of work in 4 days; A alone can do it in 12 days and B alone can do it in 18 days. How many days will be taken by C to do it alone?(a)21 (b)27 (c)9 (d)6 (e)8Q22. Which of these matrices is singularA=|8 6|  B=|-1 7|  C=|3 2|  D=|0 3|  |4 3|    |0  6|    |5 9|    |3 0| (a)A (b)B (c)C (d)D (e) NoneQ23. Match the following relationships: (i) Animal  – Lion   (1) Not a type of (ii) Television – Conveyance (2) Part of (iii) Square – Polygon  (3)  A type of (iv) Piston – Engine  (4) Superset of (a) i-3,ii-1,iii-4,iv-2 (b) i-4,ii-1,iii-3,iv-2 (c)i-4,ii-1,iii-2,iv-3 (d) i-3,ii-2,iii-1,iv-4 (e) i-1,ii-4,iii-3,iv-2Q24. If @ stands for squaring and + stands for square root what is the value of @ @ + (25)  -  + + @ (36)?(a)  625 (b)  631 (c)  619 (d) -619 (e) 627Q25. A sequence is defined recursively as  g(0) = -1; g(1) = 1  g(n) = g(n-1) + g(n-2) What will be the value of g(5)? (a)1 (b)-2 (c)3 (d)2 (e)5Q26. What curve best suits the following data: 

Page 44: Aptitude Questions

44

X Y0.99 0.0000110.04 1.0299.98 1.9971000 3.09990 4.004 (a) y = logn x (b) y = log10 x  (c) y = ex   (d) y = -log10 x  (e) y = - exQ27. A Two-dimensional array X (9,7) is stored linearly column-wise in a computer's memory. Each element requires 8 bytes for storage of the value. If the first byte address of X (1,1) is 3000, what would be the last byte address of X (8,5)?(a) 3241 (b)3541 (c)3431 (d)3556 (e)3224Q28. Of the four straight lines A,B,C,D find out which pair forms an orthogonal set  A: 3x+4y-7 = 0, B: y = -x ,  C: y = 7x+3, D: 4x = 3y+5 (a)AD (b)AC (c)BC (d)BD (e)AB Q29. Evaluate the expressionM(843,5) + R(5.8) + T(7.7) - R(3.4) whereM stands for Modular arithmetic, R stands for Round-off operation and T stands for Truncation Operation(a)12 (b)11 (c) 13 (d)14 (e)15Q30. Three independent strategies A, B and C have been initiated for cost cutting in a company producing respectively 20%, 30% and 10% savings. Assuming that they operate independently, what is the net saving achieved?(a) 45% (b) 49.6% (c) 54% (d) 56% (e) 48%Q31. The log values of two numbers to the base 10 are given as below:    X   log10X  3.142   0.4972061807  3.143   0.4973443810 Find log103.141. (a) 0.49706 (b) 0.59720 (c) 0.49110 (d) 0.49420 (e) 0.49440Q32. What equation best describes the curve shown below:(there was a curve)(a)y = tan x (b) y + 3 = x (c)y + x = 0 (d) y = ex   (e) y = Cos x Q33. The temperature at Kochi is given by the function: -t2/6+4t+12 where t is the elapsed time since noon. What is the percentage rise (or fall) in temperature between 6.00 PM and 9.00 PM? (a) 15% (b) 20% (c) 25% (d) 10% (e) 10.5%Q34. An aircraft takes off from A (72o N Lat, 40o E Long) at 2.00 AM local time to B (32o N Lat, 50o W Long). If the flying time is 8 hours what is the local time of landing at B? (a)5.40 AM (b) 3.00 AM (c) 4.00 AM (d) 5.00AM (e) 8.00AMQ35. Fill in the last row of the following Truth Table:A 0 0 0 0 1 1 1 1B 0 0 1 1 0 0 1 1C 0 1 0 1 0 1 0 1 C        B ) (A Interpret the resulting bit pattern as an integer in an 8-bit computer and write the decimal value. (a) 127 (b)129 (c)128 (d) 131 (e)122Q36. A file is transferred from one location to another in 'buckets'. The size of the bucket is 10 kilobytes. The bucket gets filled at the rate of 0.0001 kilobytes per millisecond. The transmission time from sender to receiver is 10 milliseconds per bucket. After the receipt of the bucket the receiver sends an acknowledgement that reaches sender in 100 milliseconds. Assuming no error during transmission, write a formula to calculate the time taken in seconds to successfully complete the transfer of a file of size N kilobytes. (a) 10.11 N (b) 10.011 N (c) 11.01 N (d)11.011 N (e)10.101 NQ37. The productivity of a group of workers is given by the function P(N) = 4000 √N, where N is the total strength. Find the percentage change in productivity if the strength of the group is increased by 1%. (a) 0.75%   (b) 0.5% (c) 0.25% (d) 1%   (e) 2%    Q38. A power cable is to be run from a power plant on the bank of a river 900 meters wide to a factory that is located 3000 meters downstream on the opposite bank. If the cost of laying cable under water is Rs. 5 per meter and that of laying overhead on land is Rs. 4 per meter, find the point

Page 45: Aptitude Questions

45

downstream where the cable is to cut across the river. (a) 450  (b) 3900 (c) 1800 (d) 2100 (e) 2700 

1)  Fill in the missing number in the sequence  : 26,19,17,13,11,?,8,72)  In the following series, how many Ws are there such that each W is followed by an C next to it if the C is not followed by a S next to it?(like this ,I can’t remember mine)W    C    W    S    Q    M    W    C    S    M    C    W    C    C    W    Q    M    W W    C    Q    W    C    S    C    W    A     M   C    W    C    M(a)3    (b)    5(c)6    (d)4    (e)73)  The three numbers in brackets represent the length of the sides of a triangle. Which of these does  not represent a proper triangle?(a)(2m,3m,4m)    (b)(1m,2m,4m)    (c) (3m,4m,5m) (d) (3m,3m,3m)          (e) (5m,3m,5m)    4)  My flight takes of at 2am from a place at 18N 10E and landed 10 Hrs later at a place with coordinates 36N70W. What is the local time when my plane landed?6:00 am b) 6:40am c) 7:40 d) 7:00 e) 8:00Sol) The destination place is 80 degree west to the starting place. Hence the time difference between these two places is 5 hour 20 min. (=24hr*80/360).When the flight landed, the time at the starting place is 12 noon (2 AM + 10 hours).Hence, the time at the destination place is 12 noon - 5:20 hours = 6: 40 AM5)A file is transferred from one location to another in 'buckets'. The size of the bucket is 10 kilobytes. Each bucket gets filled at the rate of 0.0001 kilobytes per millisecond. The transmission time from sender to receiver is 10 milliseconds per bucket. After the receipt of the bucket the receiver sends an acknowledgement that reaches sender in 100 milliseconds. Assuming no error during transmission, write a formula to calculate the time taken in seconds to successfully complete the transfer of a file of size N kilobytes.(n/1000)*(n/10)*10+(n/100)....as i hv calculated...~~!not 100% sure6)If G(0) = -1 G(1)= 1 and G(N)=G(N-1) - G(N-2) then what is the value of G(6)?ans: -1bcoz g(2)=g(1)-g(0)=1+1=2g(3)=1g(4)=-1g(5)=-2g(6)=-17)A, B and C are 8 bit no's. They are as follows:A -> 1 1 0 0 0 1 0 1B -> 0 0 1 1 0 0 1 1C -> 0 0 1 1 1 0 1 0 ( - =minus, u=union)Find ((A - C) u B) =?To find A-C, We will find 2's compliment of C and them add it with A,That will give us (A-C)2's compliment of C=1's compliment of C+1=11000101+1=11000110A-C=11000101+11000110=10001001Now (A-C) U B is .OR. logic operation on (A-C) and B10001001 .OR . 00110011The answer is = 10111011,Whose decimal equivalent is 1878)The size of a program is N. And the memory occupied by the program is given by M = square root of 100N. If the size of the program is increased by 1% then how much memory now occupied ?Sol) M=sqrt(100N)N is increased by 1%therefore new value of N=N + (N/100)=101N/100M=sqrt(100 * (101N/100) )Hence, we get M=sqrt(101 * N)9)In two dimensional array X(7,9) each element occupies 2 bytes of memory.If the address of first element X(1,1)is 1258 then what will be the address of the element X(5,8) ?ans.1338

Page 46: Aptitude Questions

46

10)The temperature at Mumbai is given by the function: -t2/6+4t+12 where t is the elapsed time since midnight. What is the percentage rise (or fall) in temperature between 5.00PM and 8.00PM?11)car is filled with four and half gallons of oil for full round trip. Fuel is taken 1/4 gallons more times in going than coming. What is the fuel consumed in coming up12)One rectangular plate with length 8inches, breadth 11 inches and 2 inches thickness is there. What is the length of the circular rod with diameter 8 inches and equal to volume of rectangular plate?13)what is the largest prime number in 8 digit number? (be careful bout dis type of ques  they will surely give dis type of ques) What is d largest prime number that can be stored in a 9 – bit register?Sol): 509(example)14). a)Select the odd one out a. SMTP b. WAP c. SAP d. ARPb)Select the odd one out a. Oracle b. Linux c. Ingress d. DB2c ) Select the odd one out a. WAP b. HTTP c. BAAN d. ARP15)If A, B and C are the mechanisms used separately to reduce the wastage of fuel by 30%, 20% and 10%.What will be the fuel economy if they were used combined16)Which of the following highest Standard deviation?a).5,0,5,0,5,0 b) 5,5,5,5,5,5 c) 5,5,5,5,0,5 d) -5, 5, -5, 5,-5, 5Sol): b17)A power unit is there by the bank of river of 750 meters width. A cable is made from power unit to power a plant opposite to that of the river & 1500 mts away 4m d power unit. The cost of cable below water is Rs 15/- per meter & cost of cable on the bank is Rs 12/-. Find d total cost of laying?(dis one is also very common )18)Which shape will be obtained by using these values of X & Y?X Y 0 0.00001 10 1.02 100 1.72 1000 3.00 9999 4.72 Sol): Y= log10(X) 19)1. Mammal - cow ---> a. A type of2. Snake - reptile ---> b. A part of3. Roof - Building ---> c. Not a type of4. Mushroom - Vegetables ---> d. A superset ofSol): 1-c, 2-d, 3-b, 4-a20)Find d value of @@+25 - ++@16, where @ denotes “square” and + denotes “square root”?Sol): 621(like dis)21)The size of a program is N. And the memory occupied by the program is given by M = square root of 100N. If the size of the program is increased by 1% then how much memory now occupied?Sol): 0.5%22)Find the result of the following expression- M (363, 5) +R (5.8) +T (7. 7)-R (3.8) if, M denotes modules operation, R denotes round-off, T denotes truncations?Sol): 1923)Find d unit of given expression - (momentum*velocity)/ (Acceleration*distance)?Sol): Mass24)Which is not a side of triangle?a) (2,3,6) b) (3,2,4) c) (3,4,5) d) (3,3,3) Sol): b (dis type) 25)What is the power of 4? 409626)no.of vertices,edges, faces are given.which one among these best suits for the solid planner cube. i can't remember rest of ques. their r also bar graph,pie graph ques .... for this  R>S> agarwal is enough. For QUANT some of them were quite easy such as1) Interchange “MEASUREMENTS” the first with 2nd letter, 3rd with 4th and so on., and den get the 12th letter from rightAns :) E2) There was a table where for certain values of X., there were some values of Y…Ans :) y=log(x)

Page 47: Aptitude Questions

47

3) How many zeros are there in 100!Ans ;) 24(get LCM of 100 and add the no.’s)4) the sides are given which triangle is not possible……..(A) (2,3,6)     (B) (3,2,4)     (C) (3,4,5)    (D) (3,3,3)     (E) (5,3,5)Some other Q’s were:Few were like…. 1.    58, 27, 12, __, 2 .Sol. 27*2+4 = 58,    12*2+3 = 27,  5*2+2 = 12,  2*2+1 = 5,  1*2+0 = 2 ….ans = 5 2. How the given string how many Y’s are followed by L that are not followed by PYLYPQMYLPMLYLLYQMYYLQYLPLYAMLYLM Sol. Only count those pair by YL and discard pairs of YLP      ……..ans =4. 3. First & second, third and fourth, fifth and sixth letter of word SENSATIONALY are   interchanged find the 12th letter form right.Sol.  The interchanged word is ESSNTAIONALY. Now start counting frm Right the 12th letter is E. 4. What is the largest prime number which can be stored in 6 bit register?Sol.  As the register is 6 bit long we can store max number 63 (if all bits are 1).But 63 is not prime the  largest  prime number is 61.                       Ans = 61. 5)     Number of faces, vertices and edges of a cubeAns.   6, 8, 12 6. If g(0) = 1, g(1) = -1 and g(n) = g(n-1)-g(n-2) then calculate g(6).Sol. Start with g(2) = g(1) + g(0) = -1 - 1 = -2,  g(3) = g(2) + g(1) = -2+1 = -1,  g(4) = g(3) + g(2) =  -1+1 = 0,  g(5) = g(4) + g(3) = 0+1 = 1,  g(6) = g(5) + g(4) = 1-0 = 1       ans = 1. 7. which of the following heaving highest  standard deviation.a. 9,0,9,0,9,0          b. 9,-9,0,9,0,-9   c. 9,-9,9,-9,9,-9    d. 9,9,9,9,9,9Ans.  C, standard deviation is 18(i.e. the difference between two consecutive terms is highest). 8.   Find the result of the following expression- M (737, 7) +R (3.4) +T (7. 7)-R (6.4) if, M denotes modules operation, R denotes round-off, T denotes truncations?Sol): 2+3+7-6 = 6 9. In a city, temperature at noon varies according to -t^2/6 + 4t + 12, where t is elapsed time. Find the percentage change in temperature from 3pm. to 6 pm.Sol): In equation first put t=3,we will get 25.5........................... (1)Now put t=6,we will get 42.............................. (2)So %change = ((42-25.5)/42)*100) =39.3 10.  Find d Odd one out?   a. Oracle b. Linux c. Ingress d. DB2Ans.  All other except Linux which is an operating   system  are Database Management Systems. Etc………  In a sense the university had failed. It has stores great quantities of knowledge; it teaches more people; and despite its failures, it teaches them better.__D___ . Of the great branches of knowledge- the sciences the  social sciences and humanities- the science are applied. Strenuous and occasionally successful efforts are  made to apply the social sciences, not almost never are the humanities well applied. We do not use philosophy in defining our conduct. __E__. The great task of the university in the next generation is to learn to use the  knowledge we have for the questions that come before us. __F___ The difference between a primary problem and a secondary or even tertiary problem is that primary problems tend to be around for a long time, whereasthe less important ones get solved.

One primary problem is that of interfering with biological development. ___A____. Obviously, there are benefits  both to individuals and to society from eliminating, or at least improving, mentally and physically deformed persons.  On the other hand, there could be very serious consequences if this knowledge were used with premeditation to  produce superior and subordinate classes, each genetically prepared to carry out a predetermined  mission. __B___, Here we have a primary problem that will still exist when we are all dead. Of course, the  traditional faculty members would say. “__C____”. And certainly they would not learn, but they would learn some other things.

Page 48: Aptitude Questions

48

A.    The next generation, and perhaps this one, will be able to interfere chemically with the actual development of an individual and perhaps biologically by interfering with an individual’s genes.B.    This can be done, but what happens to free will and he rights of the individualC.    But the students won’t learn enough to go to graduate schoolD.    It is in the application of this knowledge that the failure has come.E.    We do not use literature as a source of real and vicarious experience.F.    The university should organize should organize courses around primary problemsG.    The universities greatest shortcoming is not to help students see the relevance of humanities to real problems.H.    It is difficult for modern students, accustomed to the minute of film, to appreciate opera.I.    It is necessary students to require to  include in their curricula liberal arts courses.

1.) How old will C be after 10 years?Statement 1: Five years ago the average of A and B is 15 years.Statement 2: Average age of A, B and C is 20 years now.a) Statement 1 is sufficient to answer but statement 2 alone is not enough.b) Statement 2 is sufficient to answer but statement 1 alone is not enough.c) Statement 1 or statement 2 is alone sufficient to answer.d) Both statement 1 and statement 2 are not sufficient to answer the question.e) Both statement 1 and statement 2 are together sufficient to answer the question.Ans: e2.) What is the ratio between two numbers?Statement 1: The sum of two numbers is twice their difference.Statement 2: The smaller number is 6.Ans: aOptions are same for all these type of questions.3.) The sum of ages P, Q and R is 96 years. What is the age of Q?Statement 1: P is 6 years older than R.Statement 2: The total of ages of Q, R is 56 years.Ans: e4.) What is the cost price of the article?Statement 1: The profit earned is one-third of the cost price.Statement 2: The article is sold for Rs.400.Ans: e5.) A and B are in a partnership business of one year. At the end of the year, a profit ofRS.20, 000 was earned. What is A’s share?Statement 1: A invested Rs.50, 000.Statement 2: B withdrew his capital after 8 months.Ans: d6.) A and B together can complete a task in 7 days. B alone can do it in 20 days. Whatpart of the work was carried out by A?Statement 1: A completed the job after A and B worked together for 5 days.Statement 2: Part of the work done by A could have been done by B and Ctogether in 6 days.Ans: a7.) How much time did X take to reach destination?Statement 1: The ratio between the speeds of X and Y are 3:4.Statement 2: Y takes 36 minutes to reach the destination.Ans: eQuantitative2-4 Questions were asked in  the Data Sufficiency,Once read the five options given in this section because the options we knew from our books may not be the sameThese questions will be mostly from time n work,distance(trains),inequalities,ages,profit and loss If x>y,y< r options the of which then>a.x>y>z     b.z   c.z>y< P>

Page 49: Aptitude Questions

49

One more question will be from Inequalities,u have to find the limits of x or exact value of x 

2-3     questions are from probability and permutationsif there r 3 balls in red color,6 balls in green,10 balls in white,what is the probability of getting 4 balls of which 2 red and 1 white and 1 green ball?           If 5,3,2,0,1 are the five digits how many numbers of 4 digits can be formed? Venn diagrams-3 questions were asked,If  u have times material,read from those book its enough to answer these questions,Data interpretation-2questions were asked,may be a bar graph or a pie chart The three circles above represent the number of people proficient in English, Physics, Maths. Answer the next three questions based on the diagram Q14. How many more (or less) people are proficient in Physics than people who are proficient in English?(a)2      (b)3      (c)1      (d)5      (e)6

Q15. What percentages of people proficient in Maths are also proficient in English but not in Physics?(a)16    (b)19    (c)17.4 (d)12    (e)9

Q16. What percentages of total people are proficient in all three subjects?(a)9      (b)6      (c)5      (d)7      (e)8

The figure on the left represents number of distributors for a Company and the figure on the right depicts region-wise distribution for 1998 Distributors (in "00s)

Q17.  Which year has registered the maximum percentage growth in distributors?(a)1998       (b)1994       (c)1996       (d)1686       (e) 1888

Q18.  What is the average number of distributors for 1995-1999?(a) 7700      (b) 8800      (c)6800       (d)4800       (e) 7800

Q19.  If 10% of distributors in 1998 left the Company, how many fresh distributors were made in 1999?(a)4500       (b) 4600      (c)4400       (d) 4900      (e) 5400Time n work 2-3 questionsIf  a team of 5 members can do a project in 6 days,after one day one person had left,how much time will the others take to complete the project?A tap can fill a certain tank in 20ltr the other in 40ltr and the other will make it empty in 30min,whwt time they take to complete a 250ltr tank Time and distance 2-3Based on trains,A police had saw a thief running 100 mts away from him and started chasing him at aspeed of 30mts/sec and the thief is also running at 20mts/sec.how much time will the police take to catch the thiefProblems based on clocks Problems based on profit and loss,SI and CI,the price of items r given in $ and the profit is asked if u sold them at cents 2-3 questions on percentagesIf a numerator increases 10% and denominator decreases  by 20% then what is the increase or decrease percentage of this from the original number? Problems on ages,percentages,chain rule

Page 50: Aptitude Questions

50

 Rs agarwal and times books r sufficient to do this section,I got easy questions but there r another 4 sets and most of the questions will come from the above topics Critical reasoning-3 passages-12 questions-30mintwo passages  will be mostly from barrons and u can do the other passage easily,if u can do barrons passages easily its good,if not u can byheart them easily through some tips,I think u may already knew this any way I am sending u those tipsTips1) If any word like motorist appeared in the reasoning paragraph. then remember this short story : Motorist addamga(From this remind AD) velli company CEO (Ceo numchi CE gurthupettukovali) ne guddisaduso answer is :ADCEu can form the stories of your own like this to remember .this is the way I byhatted them.2)if the words marriage in the society appears in the paragraph then story is Pelli kuthurni ne cab lo ante pallaki lo (From this CAB) lo mosukuntu dum dum dum ani (From this D) thesukuvelthunnaru.So answer is:CABD3) if the word Pentagon appears in the paragraph then story is Us lo pentagon meda attack jariginappude pakkane idea cell tower vundatanta appudu adi kuda padi poyindanta attack lo.e ppudu cell lo c thesi idea lo i place lo pettandi cdea vasthundiSo answer is :CDEA     4) if the word Latin, Sanskrit appears in the paragraph then story is Dady (from this DAD) Sanskrit, latin nerchukomani books (From this remind B) thesukuvacharu        So answer is: DADB5)  if the word Baseball players appears in the paragraph then story is Base ball players addamga (From this AD)  beer ( from this BE) thagesi ishtamvachinattu aaduthunnaru         So answer is: ADBE6)  if the word Novels  appears in the paragraph then story is popular novels anni kuda bead meda (from this bead) podukuni chaduvutham kadaso the answer is :BEAD7) if the word Delegations  appears in the paragraph then story is  Delegates vachinappudu ammayalu (from this remind A) boquets echi vallaki welcome chepthu ECE department(From this ECE) ki thesukuni vellaruSo answer is : AECE8) if the word Formal Dinner  appears in the paragraph then story is  dinner lo mulakaada (drum stick from this CADA) vaddincharuSo answer is : CADA9)  if the word Visa  appears in the paragraph then story is Visa vachindi, so us velladu. us velli a to e anni chusesi ( place A first and E last) Madhya madhalo cinemalu, appudappudu college ke vellevadu(so remind CC in the middle).so answer is : ACCE10)if in the paragraph either museum or eight caves appearsanswer is :CCBB( c square b square) museum or eight caves        ccbb(c square b square)11)if in the paragraph either  memorial day appearsAnswer is: ccba (c square b a)Horizontal row, vertical row     answer:ccac(c square a c)Basket ball                             dccb(d c square b)Democratic conservative party           ddbc(d square b c)Hotel Miramar                           dbcaStory for this is in dhaba (from this DB) cat walk(from this CA) is going on  so answer is DBCAThree worker each day                   aedeSix school board members                dbdeOfficial hoster of newyork city         eadeA is the father of two children         deeb (d e square b)A,b,c,d,e ,f,q                          edbcA,b,c,w,d,e,x,f                         aabc ( a sqaure b c)http://www.ChetanaS.org

Page 51: Aptitude Questions

51

Antonyms and Synonyms: (20) (I don’t remember which were for antonyms and which were for synonym)\1. Effrontery -- extreme rudeness and lack of ability to understand that your behavior is not acceptable to other people. 2. ERRATIC(SYNONYM)a. Unromantic   b.  Free   c. Popular   d. Steady   e. Unknown(answer)3.what is the synonyms of RAPTAns:- concealed

4. SYNONYMS OF tranquilAns:- serene

5. Awry – skewed, crooked, wrong

6. Repellent x attractive(antonym)

7. Raucous – rough, wild, hoarse, guttering(synonym)

8. Benign – kind, benevolent, compassionate

9. Pristine x sullied

10. Florid= Ornate, Showy(SYN)

11. Servility= Surrender(SYN)

12. Chide * praise(ANT)

13. Stilted * natural(ANT) 

1)APTITUDE   TEST :

Questions = 82 ; time limit = 90 minutes. no negative marking. Offline (paper & pen) test and a

PSYCHOMETRY TEST also.Section 1: VERBAL    ( 32 Questions ,20 minutes )Directions for questions 1-10:Find the synonyms of the following words1. ADAGE           A. advice           B. proverb           C. enlargement           D. advantage                   Ans: B

2. TO DISPEL           A. to dissipate          B. to dissent           C. to distort           D. to disfigure           Ans: A3. ERRATIC           A. unromantic           B. unknown           C. popular           D. steady                      Ans: B   4. TO MERIT           A. to embrace           B. to devote          C. to deserve          D. to combine           Ans: C

5. RAPT           A. lively           B. concealed           C. engrossed           D. prototype                    Ans: B  6. TO HEAP           A. to pile          B. to forbid           C. to proceed           D. to share

Page 52: Aptitude Questions

52

            Ans: A  7. OVULATE           A. penury           B. immunize          C. fertilize           D. reproduce               Ans: C8. Motley          A. Uniform         B. homogenous         C.  monochrome        D. Assorted          Ans: D9.  Mitigate      A.  palliate         B. Aggravate         C. exacerbate            D. None of these         Ans: A10. Compunction      A.  Regret         B. scruple         C. qualm        D. None of these         Ans: D  Directions for questions 11-20:Find the Antonyms of the following words11. Moribund        A. declining         B. dilapidated         C.  waning        D. thriving          Ans: D 12.  Repudiate       A. reject         B. disclaim        C. acknowledge        D.  renounce,         Ans: C13.  Inundate       A. starve        B. swamp         C. deluge         D. drown        Ans: A14.  Tenacity       A. obstinacy         B. irresolution         C. firmness        D.  resolve         Ans: B15. Sobriety        A. flippancy        B. temperance         C. moderation        D.  soberness        Ans: A16. Hidebound     A. narrow-minded         B. conservative          C. prejudiced        D.  broad-minded

        Ans: D17. Nebulous       A. vague        B.  imprecise          C. precise         D. hazy          Ans: C18. Debacle       A. catastrophe        B.  fiasco        C. success        D.  shambles,        Ans: C19. Candid        A. frank         B. guarded        C. open         D. truthful        Ans: B20. Dangle         A. stick up        B.  sway          C. suspend          D. droop        Ans: ADirections for Questions 21-26: Read the passage and answer the questions that follow on the basis of the information provided in the passage.Educational planning should aim at meeting the educational needs of the entire population of all age groups.  While the traditional structure of education as a three layer hierarchy from the primary stage to the university represents the core, we should not overlook the periphery which is equally important.  Under modern conditions, workers need to rewind, or renew their enthusiasm, or strike out in a new direction, or improve their skills as much as any university professor.  The retired and the aged have their needs as well.  Educational planning, in their words, should take care of the needs of everyone.Our structures of education have been built up on the assumption that there is a terminal point to education.  This basic defect has become all the more harmful today, A UNESCO report entitled 'Learning to Be' prepared by Edger Faure and others in 1973 asserts that the education of children must prepare the future adult for various forms of self-learning.  A viable education system of the future should consist of modules with different kinds of functions serving a diversity of constituents. 

Page 53: Aptitude Questions

53

And performance, not the period of study, should be the basis for credentials.  The writing is already on the wall.In view of the fact that the significance of a commitment of lifelong learning and lifetime education is being discussed only in recent years even in educationally advanced countries, The possibility of the idea becoming an integral part of educational thinking seems to be a far cry. For, to move in that direction means such more than some simple rearrangement of the present organisation of education.  But a good begining can be made by developing open university programmes for older learners of different categories and introducing extension services in the conventional colleges and schools.  Also these institutions should learn to co-operate with the numerous community organizations such as libraries, museums, municipal recreational programmes, health services etc.21. What is the main thrust of the author ?    A) Traditional systems should be strengthened    B) Formal education is more important than non-formal    C) One should never cease to learn    D) It is impossible to meet the needs of everyone    E) There is no substitute for the extent system of education     Ans: C22. Which of the following best describes the purpose of the author ?    A) To criticise the present educational system.    B) To strengthen the present educational practices    C) To support non-conventional educational organisations.    D) To present a pragmatic point of view     E) None of these     Ans : D23.Which of the following is most nearly the same in meaning as the word 'meeting' as used in the passage.    A) Approaching    B) Contacting    C) introducing    D) representing    E) satisfying     Ans: E24. Which of the following is most opposite in meaning to the word 'integral' as used in the passage.    A) essential    B) independent    C) major    D) minor    E) unwilling     Ans: B25. According to the author, what should be the basis for awarding credentials?    A) Duration of the course    B) Competence of the course teachers    C) Diversity of the topics covered        D) Real grasp of matter or skill    E) Participation in community activities    Ans: D26. According to the author, educational plan should attempt to    A) train the people at the core    B) encourage conventional schools and colleges    C) decide a terminal point to education    D) overlook the people on the periphery    E) fulfil the educational needs of everyone.    Ans : ADirections 27-32 : Pick out the most effective word from the given words  to fill in the blank to make the sentence meaningfully complete.27. .............you meet my son in the market, ask him to come home at once    A) Will    B) While    C) Should    D) Would     Ans : C28. Sanjay was sure..............getting a first class in the examination    A) at    B) of    C) on    D) about     Ans : B29. She seems offended .................my remarks    A) with    B) for    C) upon    D) at    Ans : D30. The mounting pressure was so overwhelming that he ultimately.................to her wish    A) agreed in    B) cowed in    C) gave in    D) yielded in    Ans : C

Page 54: Aptitude Questions

54

31. He has no objection.................my proposal    A) to    B) for    C) in    D) towards    Ans: A32. Wash your hands.............water    A) from    B) with    C) by    D) in    Ans: BSection 2: QUANTITATIVE/LOGICAL REASONING ( 38 questions , 40 minutes )1.  Two Pencils 8 cents 5 Pencils cost      Ans: 20 cents2. If 2x-y=4 then 6x-3y=?    (a)15        (b)12        (c)18        (d)10

    Ans. (b)3. Pipe A can fill in 20 minutes and Pipe B in 30 mins and Pipe C can empty the same in 40 mins. If all of them work     together, find the time taken to fill the tank    (a) 17 1/7 mins        (b) 20 mins        (c) 8 mins        (d) none of these    Ans. (a)4. Thirty men take 20 days to complete a job working 9 hours a day. How many hour a day should 40 men work to     complete the job?    (a) 8 hrs        (b) 7 1/2 hrs        (c) 7 hrs        (d) 9 hrs    Ans. (b)5. A goat is tied to one corner of a square plot of side 12m by a rope 7m long. Find the area it can graze?    (a) 38.5 sq.m        (b) 155 sq.m        (c) 144 sq.m        (d) 19.25 sq.m    Ans. (a)6. Mr. Shah decided to walk down the escalator of a tube station. He found that if he  walks down 26 steps, he     requires 30 seconds to reach the bottom. However, if he steps down 34 stairs he would only require 18 seconds     to get to the bottom. If the time is measured from the moment the top step begins to descend to the time he steps     off the last step at the bottom, find out the height of the stair way in steps?    Ans.46 steps.

7. The average age of 10 members of a committee is the same as it was 4 years ago, because an old member has     been replaced by a young member. Find how much younger is the new member?    Ans.40 years.8. ABCE is an isosceles trapezoid and ACDE is a rectangle. AB = 10 and EC = 20. What is the length of AE?

    Ans. AE = 10.9.Three cards are drawn at random from an ordinary pack of cards. Find the probability that they will consist of a     king, a queen and an ace.    Ans. 64/2210.10. A number of cats got together and decided to kill between them 999919 mice. Every cat killed an equal number       of mice. Each cat killed more mice than there were cats. How any cats do you think there were ?      Ans. 99111. If Log2 x - 5 Log x + 6 = 0, then what would the value / values of x be?      Ans. x = e2 or e3.12. The square of a two digit number is divided by half the number. After 36 is added to the quotient, this sum is        then divided by 2. The digits of the resulting number are the same as those in the original number, but they are        in reverse order. The ten's place of the original number is equal to twice the difference between its

Page 55: Aptitude Questions

55

digits.               What is the  number?        Ans. 4613. A monkey starts climbing up a tree 20ft. tall. Each hour, it hops 3ft. and slips back 2ft. How much time would it       take the monkey to reach the top?      Ans.18 hours14. What is the missing number in this series?        8     2     14     6     11     ?     14     6     18     12       Ans. 915. A certain type of mixture is prepared by mixing brand A at Rs.9 a kg. with brand B at Rs.4 a kg. If the mixture       is worth Rs.7 a kg , how many kgs. of brand A are needed to make 40kgs of the mixture?      Ans. Brand A needed is 24kgs.16. A wizard named Nepo says "I am only three times my son's age. My father is 40 years more than twice my age.       Together the three of usare a mere 1240 years old." How old is Nepo?      Ans. 360 years old.17. One dog tells the other that there are two dogs in front of me. The other one also shouts that he too had two       behind him. How many are they?      Ans. Three.18. A man ate 100 bananas in five days, each day eating 6 more than the previous day. How many bananas did he       eat on the first day?      Ans. Eight.19. If it takes five minutes to boil one egg, how long will it take to boil four eggs?      Ans. Five minutes.20.Daal is now being sold at Rs. 20 a kg. During last month its rate was Rs. 16 per kg.  By how much percent      should a family reduce its consumption so as to keep the expenditure fixed?     Ans. 20 %21. Can you find out what day of the week was January 12, 1979?       Ans. Friday22. From 5 different green balls, four different blue balls and three different red balls, how many combinations of       balls can be chosen taking at least one green and one blue ball?      Ans. 3720.23. Three pipes, A, B, & C are attached to a tank. A & B can fill it in 20 & 30 minutes respectively while C can        empty it in 15 minutes. If A, B & C are kept open successively for 1 minute each, how soon will the tank be        filled?        Ans. 167 minutes.24. A person walking 5/6 of his usual rate is 40 minutes late. What is his usual time?        Ans. 3 hours 20 minutes.25. A garrison of 3300 men has provisions for 32 days, when given at a rate of 850 grams per head. At the end of         7 days reinforcement arrives and it was found that now the provisions will last 8 days less, when given at the rate       of  825 grams per head.How, many more men can it feed?       Ans. 1700 men26. In the given figure, PA and PB are tangents to the circle at A and B respectively and the chord BC is parallel to       tangent PA. If AC = 6 cm, and length of the tangent AP is 9 cm, then what is the length of the chord BC?      Ans. BC = 4 cm27. A boat travels 20 kms upstream in 6 hrs and 18 kms downstream in 4 hrs. Find the  speed of the boat in still 

Page 56: Aptitude Questions

56

       water and the speed of thewater current?     (a) 1/2 kmph        (b) 7/12 kmph        (c) 5 kmph        (d) none of these      Ans. (b)

28. Find the smallest number in a GP whose sum is 38 and product 1728     (a) 12        (b) 20        (c) 8        (d) none of these     Ans. (c)29. If x=y=2z and xyz=256 then what is the value of x?

    (a)12        (b)8        (c)16        (d)6

    Ans. (b)

30. (1/10)18 - (1/10)20 = ?    (a) 99/1020        (b) 99/10        (c) 0.9        (d) none of these    Ans. (a)31.A train running at 72 kms per hour crosses a coconut tree standing by the side of the track in 7 seconds. The      length of the train is:    (A) 104 metres        (B) 140 metres        (C) 504 metres        (D) 540 metres        Ans: B32. If 12 men and 16 boys can do a piece of work in 5 days and 13 men and 24 boys can do it in 4 days, the ratio       of daily work done by a man to that done by a boy is:    (A) 1 : 3         (B) 1 : 2        (C) 2 : 1         (D) 3 : 1    Ans: C33. A cistern is filled by a tap in 3 1/2hours. Due to a leak in its bottom, it takes 12 hour longer to fill. If cistern is       full, how long will it take to leak to empty it?        (A) 7 hours         (B) 8 hours        (C) 14 hours         (D) 28 hours        Ans: D34. A can do a piece of work in 7 days of 9 hours each whereas B can do the same work in 6 days of 7 hours each       How long will it take to complete the work together working  8  2/5 hours a day?        (A) 2 days         (B) 3 days        (C) 3 1/7 days         (D) 4 2/5 days         Ans: B35. A bus left Delhi for Ambala at 50 km/hr and turned over the same route at 40 km/hr. Thus it took 1 hour more       on the return trip. The distance between Delhi and Ambala is:        (A) 200 kms        (B) 180 kms        (C) 400 kms        (D) None of these        Ans: A36. A man purchased a bag of rice containing 70 kgs for Rs 175. He sold it at the rate of Rs 2.75 per kg.                  Find the profit and loss per cent.      (A) 10% profit         (B) 10% loss        (C) 12.5% profit         (D) 12.5% loss        Ans: A37. A shopkeeper gives 3 consecutive discounts of 10%, 15% and 15% after which he sells his goods at a           percentage profit of 30.05% on the cost price. Find the value of the percentage profit that the shopkeeper           would have earned if he had given discounts of 10% and 15% only:      (A) 53%         (B) 62.5%        (C) 72.5%         (D) 68.6%       Ans: A38. A journey of 192 kms takes 2 hours less by a fast train than by a slow train. If the average speed of the slow           train is 16 kmph less than that of a fast train, what is the average speed of the fast train?        (A) 30 kmph         (B) 48 kmph        (C) 20 kmph         (D) 25 kmph        Ans: BSection 3: CRITICAL REASONING ( 12 questions , 30 minutes)

Page 57: Aptitude Questions

57

Directions 1-12: Answer the questions given below the passage or statement as true, false or can't say.PASSAGE : In the past helicopters were forced to ground or crash because of the formation of the ice on the rotors and engines. A new electronic device has been developed which can detect the water content in the atmosphere and warns the pilot if the temperature is below freezing temperature about the formation of the ice on the rotors and wings.Answer questions 17-20 based on passage 1.The electronic device can avoid formation of the ice on the wings    Ans.False2. There will be the malfunction of rotor & engine because of formation of ice    Ans.True3. The helicopters were to be crashed or grounded    Ans.True4. There is only one device that warn about the formation of ice    Ans.TruePASSAGE : Human existence is suspicious of arbitrary divide between concise and unconcise. The concise world invades shape activity of the unconcise, while many of great activity of humanity waking as whole or partially improved by dreams. Even it could be ignored that dreams precede exceptional such a dichotomy could not be drawn as the influence of dream on waking state would remain unclear. But as yet no company rebuilt exists to record the substitute of prenatal dreaming. Answer questions 5- 8 based on passage 

5. Sleepy can be creative state     Ans. True 6. It is difficult to tell whether a sleeper is dream or not      Ans. True7. If we know what babies would dream about before they are born we could show that the concise and unconcise      mind influence on one another .     Ans. Can't say8. It is untrue claim that concise and unconcise world never impinge one another     Ans.TruePASSAGE : Although invaders represent a threat to the conservation of flora and  fauna, there are two special cases in which invasion have been deliberately brought about. One is the desire to control presents by natural predators, which may have to be brought in from other countries. The second is releasing organisms into the wild  (or on to farms, from which they might escape) that are completely novel, because they have been genetically engineered. There is nothing intrinsically sinister about engineered organisms, but any novelty must be regarded as a potential invader. Answer questions 9-12 based on passage 9.Pest control does not threat the conservation of flora and fauna.       Ans.True10.Genetically engineered organisms must always be regarded as potentially dangerous.      Ans. False 11.Natural predators are work harmful than pests.       Ans. True 12.Genetically engineered organisms escaped from the farm, they will be pose a threat to wildlife.      Ans. True

1)APTITUDE TEST:

Questions = 82 ; time limit = 90 minutes. no negative marking. Offline (paper & pen) test and a

PSYCHOMETRY TEST also.Section 1: VERBAL    ( 32 Questions ,20 minutes )Directions for questions 1-10 :Find the synonyms of the following words1. ACUMEN     A. exactness             B. potential             C. shrewdness            D. bluntness           Ans: C2. DISCRETION      A. prudence             B. consistency            C. precipice            D. disturbance          Ans: A

Page 58: Aptitude Questions

58

3. ORDAIN      A. arrange             B. command             C. contribute             D. establish                     Ans: B          4. FLORID      A. ornate            B. thriving             C. artistic             D. elegant                    Ans: A5. PENITENCE      A. liking             B. insightful             C. compunction             D. penetrable        Ans: C

6. WHET       A. stimulate             b. humorous             c. inculate             d. dampen                     Ans: A7. LATITUDE       A.  scope             B.   lamentation          C. globule             D. legislature         Ans: A8.   dispel    A. accumulate          B. collect         C.  garner         D. deploy       Ans: D9. Orthodox    A. heterodox        B. unconventional        C. conventional         D. untraditional       Ans: C10. Alienate     A. be friendly         B. estrange       C. disarm        D. None of these        Ans: BDirections for questions 11-20:Find the Antonyms of the following words11 .Dogmatic     A. Arbitrary         B. doctrinal         C. unbending        D. Flexible     Ans: D12. Fallible     A. Perfect        B. Imperfect         C.  unsound         D. mortal      Ans: A13. Harbinger    A. Forerunner        B. herald        C. potent        D.  None of these        Ans: D14. Intermittent    A. Alternating         B.  sporadic        C. Constant        D. None of these     Ans: C15. Pensive    A. ignorant        B. Brooding         C. pondering         D. meditative        Ans: A16. Concur     A. harmonize         B. coincide         C. assent         D. Conflict        Ans: D17.Furtive   A. Open        B. Surreptitious          C. clandestine        D.  None of these        Ans: B18. Efface    A. Obliterate         B. Engrave        C. eradicate        D. None of these        Ans: B19.  Pretentious     A. Ostentatious         B. pompous         C. Down-to-earth        D. conceited              Ans: C20. Veer       A. avert          B. skew        C. whirl            Dstay

Page 59: Aptitude Questions

59

       Ans: DDirections for Questions 21-26: Read the passage and answer the questions that follow on the basis of the information provided in the passage.Amnesty International's charge that 'tens of thousands' of political prisoners, including prisoners of conscience, are languishing in Indian jails and that prisoners are routinely tortured in this country has to be seen in a much wider context than the organisation's annual report cares to do.  In its overall appraisal of 151 countries, Amnesty has accused 112 of torturing prisoners, 63 of harboring prisoners of conscience, 61 of resorting to political killings and 53 of detaining people without a trial.  Of these apparently overlapping  categories, India seems to have been excluded from the list of the 61 which undertake political killings.  The report has however, pointed out that scores of people in India die of torture in police and military custody and that many also simply disappear.  Clearly, only a thin line separates the 61 charged with political murder from the rest.  Before coming to such conclusions, however, it may also be necessary to classify the various countries according to their political systems. Torture by the security forces and killings at the behest of the government  make no difference to the victims whether they are in a democratic country or a totalitarian one. It is also nobody's case that a democratic country is less culpable than a dictatorship in the event of human rights violations. But the point perhaps still needs to be made that torture or 'disappearances' represent a failure of the system in a democracy in contrast to being an integral part of state policy in a country ruled by an autocrat who is answerable to no one.India may be guilty of keeping 'tens of thousands' behind bars and of the other human rights abuses mentioned by qualitatively different place from a totalitarian country.  It is in this respect that Amnesty has been lass than fair.  It has chosen to ignore the distinctions between the good, the bad and the ugly.  The openness of Indian society will be evident to anyone who spends half an hour in one of its chaotic market-places or visits the law courts or watches a political rally or reads a newspaper or strikes  up a conversation with any person on the roads.  There is no sense of fear in India, as in a dictatorship.  There is also scope for securing relief from the heavy-handed behaviour of the authorities, even if the human rights commission has not yet lived up to expectations.  Unless such points are recognised, Amnesty's assessment will seem to be a dry recital of statistics which may pillory India simply because of its larger population.  Mercifully, Amnesty nowadays at least notes that the terrorists also indulge in human rights violations and that India has to cope with several insurgencies fomented by a country where the military does not always seem to be under the control of the elected government.  True, there is much that is the way the terrorist challenge is activating the self-correcting mechanism within a democracy and not merely on painting a grim, even biased picture.21. In the report, India has been excluded from which of the following categories of violating human rights?    A) Torturing prisoners    B) Detaining without trial    C) Political killings    D) harbouring prisoners of conscience    E) None of these     Ans: C22. Which of the following is not true in the context of the passage ?    A) India is guilty of some human rights abuses    B) Amnesty International appraised all the democratic countries    C) There is overlapping of cases in the categories of human right abuses    D) India was one of the countries appraised by Amnesty International    E) The report notes that the terrorists also violate human rights     Ans: B23. According to the passage, through which media of forum Amnesty International has hurled the charges?    A) Seminar on Human rights    B) Its Regional Meet    C) Its annual Report    D) Its International Meet    E) None of these     Ans: C24. The author of the passage    A) agrees with the report    B) disagrees with the report

Page 60: Aptitude Questions

60

    C) disagrees that conditions of prisons in India is bad    D) supports the totalitarian approach    E) disagrees with report on terrorists     Ans: B25. The Amnesty Internationals report is based on the information of how many countries ?    A) 63    B) 112    C) 131    D) 115    E) None of these     Ans: E26. The author suggests classification of various countries on the additional dimension.  Which of the following is that dimension ?    A) Economic progress            B) Human rights            C) Industrial Progress    D) Political systems                E) None of these    Ans: BDirections 27-32 : Pick out the most effective word from the given words  to fill in the blank to make the sentence meaningfully complete.27. The boy you met yesterday is in class..................       A) ninth    B) the ninth    C) nine    D) the nine         Ans : C28. There was some confusion.................the agreement       A) on    B) in    C) around    D) over        Ans : D29. Speak loudly as he is slow.........................hearing    A) in     B) about    C) at    D) of     Ans : C30. Will you..................may dog while I am on tour ?    A) look out    B) look up    C) look on    D) look after     Ans : D31. Savitha is disgusted...................the habits of her husband    A) of    B) from    C) with    D) at     Ans : C32. Every man craves....................recognition    A) for    B) about    C) at    D) after     Ans : DSection 2: QUANTITATIVE/LOGICAL REASONING ( 38 questions , 40 minutes )1. If a + b + c = 0, then (a3 + b3 + c3) ÷ abc is equal to:    (A) 1         (B) 2        (C) 3           (D) 9    Ans : C2.The difference between compound interest and simple interest for 3 years at 5% per annum can be found out by     multiplying the principal by:     (A) 1.7625         (B) 0.7625        (C) 0.07625         (D) 0.007625     Ans : A3.The simple interest on a sum of money is 1/9 th of the sum and the number of years and the rate per cent per     annum are equal. The rate per cent per annum is:    (A) 3 1/3         (B) 5        (C) 6 2/3         (D) 10    Ans : A4.A man invested Rs 5,000 at some rate of simple interest and Rs 4,000 at 1% higher rate of interest. If the interest in both the cases after 4 years is same the rate of interest in the former case is: (A) 4%         (B) 5%        (C) 6 1/4%         (D) 8  1/3 %    Ans : A5. A sum of money at simple interest rate amounts to Rs 4,025 in 3 years and to Rs 4,550 in 6 years at the same     rate of interest. Find the sum and the rate of interest per annum.    (A) Rs 2,500, 6%         (B) Rs 3,000, 5%        (C) Rs 3,500, 5%         (D) Rs 4,500, 4%    Ans : C6. On a certain map of India the actual distance of 1450 kms between two cities Delhi and Kolkata is shown as          5 cms. What scale is used to draw the map?    (A) 1 : 15 × 106         (B) 1 : 20 × 106        (C) 1 : 25 × 106         (D) 1 : 29 × 106

Page 61: Aptitude Questions

61

    Ans : D7. The ratio between the third proportional of 12 and 30 and mean proportional of 9 and 25 is:        (A) 2 : 1         (B) 5 : 1        (C) 7 : 15         (D) 9 : 14        Ans : B8. Rs 1,050 are divided among P, Q and R. The share of P is 2/5 of the combined share of Q and R. P gets:     (A) Rs 320         (B) Rs 300        (C) Rs 200         (D) Rs 420    Ans : B                                    ma + nc9. If a : b = c : d, then ________      is equal to:                                   mb + nd    (A) an : mb         (B) m : n         (C) a : b         (D) dm : cn        Ans : C10. How many even numbers of four-digits can be formed with digits 1, 2, 3, 4, 5, 6 (repetition of the digit is       allowed)?        (A) 648         (B) 180        (C) 1296         (D) 540       Ans : A11. There is a number lock with four rings. How many attempts at the maximum would have to be made before        getting the right number?       (A) 104         (B) 255         (C) 104 – 1         (D) 256       Ans : C12. There are four letters and four envelops addressed to different persons. In how many ways can wrong choices        be made?    (A) 64         (B) 23         (C) 16         (D) 255        Ans : D13. There are 10 points on a straight line AB and 8 points on another AC, none of them being A. How many        triangles can be formed with these points as vertices?        (A) 720         (B) 640         (C) 816         (D) 680         Ans : B14. A circular ground of circumference of 88m. A strip of land 3m wide inside and along the circumference is to be       leveled. What is the expenditure if leveling cost is Rs. 7 per metre square.       Ans. 165015. There are 4 boys and 3 girls. What is the probability the boys and girls sit alternately?       Ans 1/3516. Two trains are 2 kms apart. Speed of one train is 20m/s and the other train is running at 30 m/s . Lengths of the       trains are 200 and 300m. In how much time do the trains cross each other?       Ans. 50 seconds17. A& B are two players. They need to select one number from 1 to 25. If both the players select the same        numbers they will win the prize. What is the probability of not winning in a single trial?        Ans 24/2518. Four different integers are in increasing AP such that one number is equal to sum of the squares of the other        three numbers.      Ans –1,0,1,219. In a company there were 75 % skilled employees and the remaining unskilled. 80% of the skilled and 20% of       the unskilled were permanent. If the temporary employees were 126 find the total number of employees.      Ans 360 20. A person sells a horse at 12.5% loss. If he sells for 92.5 more, he will have a profit of 6%. What is the CP?      Ans 500

Page 62: Aptitude Questions

62

21. One tap takes 15 min to fill, another tap takes 12 min to fill and the third tap can empty in 20 min. In how much       time the tank would be full.      Ans 10 min22. Two trains are separated by 200km. One leaves at 6:00 am from Delhi and reaches Merrut at 10:00 am.       Another train leaves from Merrut at 8:00 am and reaches Delhi at 11:30 am. At what time two trains meet each       other?    Ans 8:56 am23. If the height of the triangle decreases by 40% and the breadth increases by 40%. Then the effect on the area is      Ans. 16% decrease.24. A grandfather has 5 sons and daughters and 8 grandchildren.. They have to be arranged in arrow such that the       first 4 seats and last four seats are to be taken by grandchildren and the grandfather would not sit adjacent to      any of the grandchildren.        Ans. 4 * 8! * 5!25. A farmer has a rectangular plot. He wants to do fencing along one of the side with the help of the posts. Two       posts being on two corners. He brings 5 post less than what he has initially plan  because of which the distance       between two consecutive post became 8 m instead of 6 m.. What is the length of the side and no of post?        Ans. 120 , 1626. A circular ground of circumference of 88m. A strip of land 3m wide inside and along the circumference is to be       leveled. What is the expenditure if leveling cost is Rs. 7 per metre square ?      Ans. 165027. Four horses are tethered at the four corners of a square of side 14cm such that two horses along the same side       can just reach each other. They were able to graze the area in 11 days. How many days will they take in order       to graze the left out area?      Ans. 328. Six bells commence tolling together and toll at intervals 2,4,6,8,10 and 12 seconds respectively. In 30 minutes        how many times they toll together.        a) 4        b) 10        c) 15        d) 16           Ans:  d)29. The number of coins 1.5 cm in diameter and 0.2cm thick to be melted to form a right circular cylinder of height       10 cm and diameter 4.5 cm is:        a) 380        b) 450        c) 472        d) 540        Ans:  b)30. The size of a wooden block is 5 * 10 * 20 cm3. How many whole such blocks you will take to construct a solid      wooden cube of minimum size?        a) 6        b) 8        c) 12        d) 16        Ans:  b)31. Find the odd man out - 1050, 510, 242, 106, 46, 16, 3    a) 510        b) 242        c) 106            d) 46        Ans:  C)32. A clock loses 10 minutes each hour. If the clock is set correctly at noon, what time is it when it reads 3 PM?        Ans: 8 minutes.33. If 8 crows can steal 8 buttons in 8 minutes, how long will it take 16 crows to steal 16 buttons?      32 buttons

Page 63: Aptitude Questions

63

34. The amount of water in a tank doubles every minute. The tank is full in an hour. When was the tank half full?        Ans: 59 minutes.35. A laborer can dig a hole 8 feet square and 8 feet deep in 8 days. How long will it take him to dig a hole 4 feet      square and 4 feet deep?        Ans: 1 day36. A clock loses 10 minutes each hour. If the clock is set correctly at noon, what time is it when it reads 3 PM?      Ans: 3:36 PM.37. If 2 miles of fence enclose a square plot of 160 acres, how large a square will 4 miles of fence enclose?     Ans: 640 acres or 1 square mile.38. A woman travels 1 mile south, then one mile west, and then mile north, and arrives at her starting point.          Where is she?    Ans: NorthSection 3: CRITICAL REASONING ( 12 questions , 30 minutes)Directions1-12: Answer the questions given below the passage or statement as true, false or can't say.PASSAGE  : Copernicus is the intelligent. In the days of Copernicus the transport and technology development was less & it took place weeks to communicate a message at that time, wherein we can send it through satellite with in no time. Even with this fast developments it has become difficult to understand each other.Answer questions 1-4 based on passage  .1. People were not intelligent during Copernicus days        Ans. False2. Transport facilities are very much improved in noe a days        Ans. Can't say3. Even with the fast developments of the technology we can't live happily.        Ans. Can't say4. We can understand the people very much with the development of communication        Ans. False.3. Critical Reasoning ( 12 questions , 30 minutes) PASSAGE: Senior managers warned the workers that because of the introductory of Japanese industry in the car market. There is the threat to the workers. They also said that there will be the reduction in the purchase of the sales of car in public. the interest   rates  of the car will be increased with the loss in demand.Answer questions 5-8 based on passage .5. Japanese workers are taking over the jobs of Indian industry.    Ans. False6.Managers said car interests will go down after seeing the raise in interest rates.    Ans. True7. Japanese investments are ceasing to end in the car industry.    Ans. False8. People are very interested to buy the cars.    Ans. FalsePASSAGE: There should be copyright for all arts. The reels has came that all the arts has come under one copy right society, they were use the money that come from the arts for the developments . There may be a lot of money will come from the Tagore works. We have to ask the benifiters from Tagore work to help for the development of his works.Answer questions 35-39 based on passage .9. Tagore works are came under this copy right rule.        Ans. False10. People are free to go to the public because of the copy right rule.        Ans. Can't say

11. People gives to theater and collect the money for development.        Ans. Can't say

Page 64: Aptitude Questions

64

12. We have  ask the Tagore residents to help for the developments of art.

       Ans.Can't say1) APTITUDE TEST:

Questions=82; Time limit=90 minutes. No negative marking. Offline (paper & pen) test and a

Psychometry test also.Section 1: VERBAL ( 32 Questions, 20 minutes )Directions for questions 1-10: Find the synonyms of the following words1. POTENTIAL       a. latent             b. hysterical             c. conventional             d. symmetrical   Ans: a        

2. EXTRICATE         a. terminate             b. isolate             c. liberate             d. simplify     Ans: c          3. DISPARITY       a. inequality             b. impartiality             c. unfairness             d. twist         Ans: a       

4. TO CONFISCATE     a. to harass             b. to repulse             c. to console             d. to appropriateAns: d         5. PIOUS        a. historic             b. devout             c. multiple             d. fortunate    Ans: b          

6. CARGO         a. cabbage             b. camel             c. lance             d. freightAns: d     7. OVATION      a. oration             b. gesture             c. emulation              d. applause   Ans: d              8.  Candid     a. intriguing        b. sly        c. frank          d. cunningAns: c9.  Murky     a. overcast        b. dazzling        c. Clear         d. fulguranAns: a10. Guile     a. Openness        b. slyness        c. fair-dealing        d. ingenuousnessAns: bDirections for questions 11-20: Find the Antonyms of the following words11. Stern       a. Lenient        b. Crabby       c.  Tenant       d. UnreasonableAns: a12. Spry         a. agile        b. young       c.  adult        d. dodderingAns: d14. Spurn         a. weave        b. turn       c. sew         d. embraceAns: d15. Slothful          A. summit       B. animal      C. lazy     D. industriousAns: D16. Sluggish        A. wasteful      B. brisk      C. baseball     D. chillyAns: B17. Scrawny              A. bane     B. skinny      C. obese     D. lean

Page 65: Aptitude Questions

65

Ans: C18. Tolerance     A. intolerance         B. forbearance          C. lenience         D.  acceptanceAns: A19. Susceptible        A.  sensitive         B. swayable         C. amenable        D. imperviousAns: D20. Stingy         A. miserly         B. generous         C. parsimonious         D. sparingAns: BDirections for Questions 21-26: Read the passage and answer the questions that follow on the basis of the information provided in the passage.Recent advances in science and technology have made it possible for geneticists to to find out abnormalities in the unborn foetus and take remedial action to rectify some defects which would otherwise prove to be fatal to the child. Though genetic engineering is still at its infancy, scientists can now predict with greater accuracy a genetic disorder.  It is not yet in a position to predict when exactly a genetic disorder will set in.  While they have not yet been able to change the genetic order of the gene in germs, they are optimistic and are holding out  that in the near future they might be successful in achieving this feat.  They have, however, acquired the ability in manipulating tissue cells.  However, genetic mis-information can sometimes be damaging for it may adversely affect people psychologically.  Genetic information may lead to a tendency to brand some people as inferiors.  Genetic information can therefore be abused and its application in deciding the sex of the foetus and its subsequent abortion is now hotly debated on ethical lines. But on this issue geneticists cannot besquarely  blamed though this charge has often been levelled at them. It is mainly a societal problem.  At present genetic engineering is a costly process of detecting disorders but scientists hope to reduce the costs when technology becomes more advanced.  This is why much progress in this area has been possible in scientifically advanced and rich countries like the U.S.A, U.K and Japan.  It remains to be seen if in the future this science will lead to the development of a race of supermen or will be able to obliterate  disease from this world.21. Which of the following is the same in meaning as the phrase 'holding out' as used in the passage    A) catching    B) expounding    C) sustaining    D) restraining    E) controlling Ans: B22. According to the passage, the question of abortion is    A) ignored    B) hotly debated    C) unanswered    D) left to the scientists to decide    E) already settled Ans: B23. Which of the following is the same in meaning as the word 'obliterate' as used in the passage.    A) wipe off    B) eradicate    C) give birth to     D) wipe out    E) very literate Ans: B24. Which of the following is the opposite in meaning to the word 'charged' as used in the passage    A) calm    B) disturbed    C) discharged    D) settled    E) peaceful Ans: D25. Which of the following is not true of the genetic engineering movement.    A) Possibility of abuse    B) It is confronted by ethical problems    C) Increased tendency to manipulate gene cells    D) acquired ability to detect genetic disorders in unborn babies.    E) acquired ability to manipulate tissue cells Ans: C26. Which of the following is the same in meaning as the word 'feat' as used in the passage    A) process    B) focus    C) fact    D) possibility    E) goalAns: EDirections 27-32: Pick out the most effective word from the given words  to fill in the blank to make the sentence meaningfully complete.27. He preferred cricket.....................any other sport.    A) against    B) to    C) over    D) thanAns: B

Page 66: Aptitude Questions

66

28. There is a disturbing nexus.....................politics and sports in India    A) between    B) with    C) among    D) byAns: A29. Don't loiter.................the street    A) in    B) about    C) on    D) intoAns: B30. The terms given to you are not acceptable ..............me    A) for    B) by    C) with    D) toAns: D31. You have played a great role, for ..................your help I possibly would have landed myself into a problem.    A) after    B) despite    C) unless     D) although    E) withoutAns: E32. Fate smiled..............him in all his ventures    A) upon    B) on    C) at    D) overAns: BSection 2: QUANTITATIVE/LOGICAL REASONING ( 38 questions, 40 minutes )1. A pupil's marks were wrongly entered as 83 instead of 63. Due to that the average marks for the class got increased by half. The number of pupils in the class is    A) 10    B) 20    C) 40    D) 73Ans: C2. The average weight of A, B and C is 45 kg.  If the average weight of A and B be 40 kg and that of B and C  be 43 kg, then the weight of B is    A) 17 kg    B) 20 kg    C) 26 kg    D) 31 kgAns: D3. The average monthly salary of 20 employees in an organisation is Rs. 1500.  If the manager's salary is added, then  the average salary increases by Rs. 100. What is the manager's salary?     A) Rs.2000    R) Rs.2400    C) Rs. 3600    D) Rs.4800Ans: C4. The average of five consecutive numbers in n. If the next two numbers are also included, the average will :    A) remain the same   B) increase by 1    C) increase by 1.4     D) increase by 2Ans: B5. In an election between two candidates, one got 55% of the total valid votes, 20% of the votes were invalid. If the total number of votes was 7500, the number of valid votes that the other candidate got, was    A) 2700    B) 2900    C) 3000    D) 3100Ans: A6. Two tailors X and Y are paid a total of Rs. 550 pe week by their employer. If X is paid 120 percent of the sum paid to Y, how much is Y paid per week    A) Rs. 200    B) Rs. 250    C) Rs.300    C) Rs. None of theseAns : B7. A student secures 90%, 60% and 54% marks in test papers with 100, 150 and 200 respectively as maximum marks. The percentage of his aggregate is    A) 64    B) 68    C) 70    D) None of theseAns: A8. A man gains 20% by selling an article for a certain price.  If he sells it at double the price, the percentage of profit will be    A) 40    B) 100    C) 120    D) 140Ans: D9. Profit earned by selling an article for Ra. 1060 is 20% more than the loss incurred by selling the article for Rs. 950.  At what price should be article be sold to earn 20% profit.?     A) 980    B) 1080    C) 1800    D) None of theseAns: D10. The ages of A and B are in the ration of 3:1. Fifteen years hence, the ratio will be 2:1 Their present ages are     A) 30 years, 10 years    B) 45 years, 15 years    C) 21 years, 7 years    D) 60 years, 20 yearsAns: B

Page 67: Aptitude Questions

67

11. The electricity bill of a certain establishment is partly fixed and partly varies as the number of units of electricity consumed.  When in a certain month 540 units are consumed, the bill is Rs. 1800. In another month 620 units are consumed and the bill is Rs. 2040.  In yet another month 500 units are consumed.  The bill for that month would be      A) Rs.1560    B) Rs. 1680    C) Rs. 1840    D) Rs. 1950Ans: B12. A and B are two alloys of gold and copper prepared by mixing metals in the ratio 7: 2 and 7: 11 respectively. If equal quantities of the alloys are melted to form a third alloy C, the ratio of gold and copper in C will be      A) 5:7    B) 5:9    C) 7:5    D) 9:5Ans: C13. Three men, four women and six children can complete a work in seven days.  A women does double the work a man does and a child does half the work a man does.  How many women alone can complete this work in 7 days      A) 7    B) 8    C) 12    D) Cannot be determined    E) None of theseAns: A14. A man, a women and a boy can complete a job in 3, 4 and 12 days respectively.  How many boys must assist 1 man and 1 women to complete the job in 1/4 of a day.       A) 1    B) 4    C) 19    D) 41Ans: D15. If 6 men and 8 boys can do a piece of work in 10 days while 26 men and 48 boys can do the same in 12 days, the time taken by 15 men and 20 boys in doing the same type of work will be :     A) 4 days    B) 5 days    C) 6 days    D) 7 daysAns: A 16. With a uniform speed a car covers the distance in 8 hours.  Had the speed been increased by 4 km/hr, the same distance could have been covered in 7/1/2 hours.  What is the distance covered?    A) 420 km    B) 480 km    C) 640 km    D) Cannot determined    E) None of theseAns: B17. In a flight of 600 km, a aircraft was slowed down due to bad weather.  Its average speed for the trip was reduced by 200 km/ hr and the time of flight increased by 30 minutes.  The duration of the flight is:     A) 1 hour    B) 2 hours    C) 3 hours    D) 4 hoursAns: A18. It takes eight hours for a 600 km journey, if 120 km is done by train and the rest by car. It takes 20  minutes more, if 200 km is done by trai  and the rest by car.  The ratio of the speed of the train to that of the car is :    A) 2 : 3    B) 3 : 2    C) 3 : 4    D) 4 : 3Ans: C19. A goods train runs at the speed of 72 kmph and crosses a 250 m long platform in 26 seconds.  What is the length of the goods train ?     A) 230 m    B) 240 m    C) 260 m    D) 270 m Ans: D20. A 300 metre long train crosses platform in 39 seconds while it crosses a signal pole in 18 seconds.  What is the length of the platform ?    A) 320 m    B) 350 m    C) 650 m    D) Data inadequate    E) None of theseAns: B21. A 270 metres long train running at the speed of 120 kmph crosses another train running in opposite direction at the speed of 80 kmph in 9 seconds. What is the length of the other train    A) 230 m    B) 240 m    C) 260 m    D) 320 m     E) None of theseAns: A22. A man can row at 5 kmph in still water. If the velocity of current is 1 kmph and it takes him 1 hour to row to a place and come back, how far is the place ?    A) 2.4 km    B) 2.5 km    C) 3 km    D) 3.6 kmAns: A23. A boat takes 19 hours for travelling downstream from point  A to point B and coming back to a point C midway between A and B. If the velocity of the stream is 4 kmph and the speed of the boat in still water is 14 kmph, what is the distance between A and B ?      A) 160 km    B) 180 km    C) 200 km    D) 220 kmAns : B

Page 68: Aptitude Questions

68

24. Peter invested an amount of Rs. 12000 at the rate of 10 p.c.p.a. simple interest and another amount at the rate of 20 p.c.p.a simple interest.  The total interest earned at the end of one year on the total amount invested become 14 p.c.p.a. Find the total amount invested.     A) 20,000    B) 22,000    C) 24,000    D) 25,000    E) None of theseAns: A25. The rates of simple interest in two banks A and B are in the ratio 5 : 4. A person wants to deposit his total savings in two banks in such a way that he received equal half-yearly interest from both. he should deposit the savings in banks A and B in the ration :     A) 2 : 5        B) 4 : 5    C) 5 : 2    D) 5 : 4Ans : B26. What will be the cost of gardening 1 metre broad boundary around a rectangular plot having perimeter of 340 metres at the rate of Rs.10 per square metre?    A) Rs. 1700    B) Rs. 3400    C) Rs. 3440      D) Cannot be determined    E) None of theseAns: C27. The length and breadth of the floor of the room are 20 feet and 10 feet respectively. Square tiles of  2 feet length of different colours are to be laid on the floor. Black tiles are laid in the first row on all sides. If white tiles are laid in the one-third of the remaining and blue tiles in the rest, how many blue tiles will be there?    A) 16    B) 24    C) 32    D) 48    E) one of theseAns: A28. A park  square in shape has a 3 metre wide road inside it running along its sides. To area occupied by the road is 1764 square metres. What is the perimeter along to outer edge of the road?    A) 576 metres    B) 600 metres    C) 640 metres    D) Data inadequate    E) None of theseAns: B29. At what time between 7 and 8 o'clock will the hands of a clock be in the same straight line but not together    A) 5 min. past 7    B) 5 2/11 min. past 7    C) 5 3/11 min. past 7    D) 5 5/11 min. past 7Ans: D30. A watch which gains uniformly is 2 minutes low at noon on Monday and is 4 min. 48 sec fast at 2 p.m on the following Monday. When was it correct ?    A) 2 p.m on Tuesday    B) 2 p.m on Wednesday    C) 3 p.m on Thursday    D) 1 p.m on FridayAns: B31. How many shares of market   value  Rs. 25 each can be purchased for Rs. 12750, brockerage being 2 % ?    A) 450    B) 500    C) 550    D) 600Ans : B32. A Man buys Rs. 50 shares in a company which pays 10% dividend.  If the man gets 12.5% on his investment, at  what price did he buy the shares ?    A) Rs. 37.50    B) Rs. 40    C) Rs. 48    D) Rs. 52Ans: B33. In how many different ways can be letters of the word, 'RUMOUR' be arranged ?    A) 180    B) 90    C) 30    D) 720    E) None of theseAns: A34. In how many different ways can the letters of the word 'AUCTION' be arranged in such a way that the vowels always come together ?    A) 30    B) 48    C) 144    D) 576    E) none of theseAns: D35. In a group of 6 boys and 4 girls, four children are to be selected.  In how many different ways can they be selected such that at least one boy should be there ?    A) 159    B) 194    C) 205    D) 209    E) None of theseAns: D36. A box contains 5 green, 4 yellow and 3 white marbles. Three marbles are drawn at randum. What is the probability that they are not of the same colour ?    A) 3/44    B) 3/55    C) 52/55    D) 41/44Ans: D37. In a box, there are 8 red, 7 blue and 6 green balls. One ball is picked up randomly. what is the probability that it is neither red nor green ?    A) 2/3    B) 3/4    C) 7/19    D) 8/21    E) 9/21

Page 69: Aptitude Questions

69

Ans : D38. The true discount on Rs. 1760 due after a certain time at 12% per annum is Rs. 160. The time after which it is due is     A) 6 months    B) 8 months    C) 9 months    D) 10 monthsAns: DSection 3: Critical reasoning (12 questions, 30 minutes)Directions 1-12: Answer the questions given below the passage or statement as true, false or can't say.Passage: Glaciers begin to form where snow remains year-round and enough of it accumulates to transform into ice. New layers of snow compress the previous layers and this compression forces the icy snow to re-crystallize, forming grains similar in size and shape to cane sugar. Gradually the grains grow larger and the air pockets between the grains get smaller, meaning that the snow slowly becomes more dense. After about two winters, the snow turns into firn, an intermediate state between snow and ice. Over time the larger ice crystals become more compressed and even denser, this is known as glacial ice. Glacial ice, because of its density and ice crystals, often takes a bluish or even green hue.Answer questions 1- 5 based on passage.1. Glaciers cannot form where snow does not remain all year round.           A. True                B. False                C. Can't sayAns: A2. Firn is less dense than snow but more dense than ice          A. True                B. False                C. Can't sayAns: B3. Glacial ice is always greenish or bluish in color.         A. True                B. False                C. Can't sayAns: B4. Snow falls every year in areas where glaciers form.        A. True                B. False                C. Can't sayAns: C5. The increase in density is caused by the grains becoming smaller.          A. True                B. False                C. Can't sayAns: BPassage: A power of attorney or letter of attorney in common law systems or mandate in civil law systems is an authorization to act on someone else's behalf in a legal or business matter. The person authorizing the other to act is the "principal" or "grantor", and the one authorized to act is the "agent" or "attorney-in-fact". The attorney-in-fact acts "in the principal's name," signing the principal's name to documents and filing suit with the principal's name as plaintiff, for example.As one kind of agent, an attorney-in-fact is a fiduciary for the principal, so the law requires an attorney-in-fact to be completely honest with and loyal to the principal in their dealings with each other. If the attorney-in-fact is being paid to act for the principal, the contract is a separate matter from the power of attorney itself, so if that contract is in writing, it is a separate document, kept private between them, whereas the power of attorney is intended to be shown to various other people.The power of attorney may be oral, such as asking someone else to sign your name on a cheque because your arm is broken, or may be in writing. Many institutions, such as hospitals, banks, and the I.R.S., require a power of attorney to be in writing before they will honor it, and they usually want to keep an original for their records.Answer questions 6-10 based on passage.6. The agent grants the principal the power to act on behalf of the grantor.    A. True                B. False                C. Can't sayAns: B7. All contracts between the principal and the agent must be made public.    A. True                B. False                C. Can't sayAns: B8. The power of attorney may be granted verbally.    A. True                B. False                C. Can't sayAns: A9. Only a legal professional can be granted the power of attorney.    A. True                B. False                C. Can't sayAns: C

Page 70: Aptitude Questions

70

10. In civil law systems the power of attorney is referred to as a mandate.    A. True                B. False                C. Can't sayAns: APASSAGE :Any one who has has systematic exam phases will have perceived a profound although not a prolif of asymmetry whether or not the exception is volitions and self control of spontaneous appeal to predict facial as symmetry as does the type of emotion portrayed. position can not displace symmetric at left side regret of a negative emotion is more common posed expression negation emotions are likely to be symmetric representation and where as symmetric occurs relative left sided expression is more common. Answer questions 11-12 based on passage .

11.  Any angry person is more likely to have left sided expression than some one who has smiling          A. True                B. False                C. Can't sayAns: A12.  An actor is likely to smile symmetric when acting         A. True                B. False                C. Can't sayAns: B

1)APTITUDE TEST:

Questions = 82 ; time limit = 90 minutes. no negative marking. Offline (paper & pen) test and a

PSYCHOMETRY TEST also.Section 1: VERBAL    ( 32 Questions ,20 minutes )Directions for questions 1-10:Find the synonyms of the following words 1.     Diligence        A. Laziness        B. carelessness        C.  negligence        D. meticulousness         Ans: D2. Illustrious         A. unimportant        B. glorious        C.  shameful       D. None of these        Ans: B3.  prolific       A. Unproductive         B. scarce           C. abundant           D. None of these       Ans: C   4.  Relinquish       A.  Vanquish  B. repudiate        C.  retain,        D. None of these       Ans: B5.   ABODE      A. clay             B. obstacle             C. dwelling             D. bind        Ans: C             6.   BEHEST      A. behavior             B. hold down          C. relieve            D. condemn       Ans: B7.  MORTIFY     A. make a cavity           B. displease           C. humiliate           D. relapse       Ans: C             8.  LETHARGY      A. reminiscence             B. category             C. fallacy             D. stupor     Ans: D 9.    INCENTIVE       A. provocation              B. amplitude             d. reflex          e. escutcheon      Ans: A 10. CAJOLE        a.   mollify         b. motivate           c. profound           d.  coax          Ans: D Directions for questions 11-20:Find the Antonyms of the following words11.  admonish        A. Reprove        B. reprimand        C.  Approve        D.  rebuke        Ans: C12.  Circumspect       A.  Reckless         B. Cautious        C. prudent        D.  guarded

Page 71: Aptitude Questions

71

        Ans: A13.    Impetus        A.  Momentum       B. thrust      C. motivation       D. Inertia        Ans: D14.     Latent       A.  Dormant      B. Overt      C. embryonic       D.  suppressed        Ans: B15.   Manifestation        A. demonstration        B. expression       C. obscurity        D. symptom         Ans: C16.    Solicit      A.  Importune        B. implore        C. crave           D. Grant      Ans: D17.  Vehement      A. Fervent        B. passionate        C. Apathetic        D. None of these        Ans: C18.  Caprice     A. Whim         B. impulse         C. quirk        D. None of these     Ans: D19. Dwindle       A. extend         B. fall         C. fall off         D. drop        Ans: A20. Covet       A. aspire to         B. abjure        C. begrudge         D. crave      Ans: BDirections for Questions 21-26: Read the passage and answer the questions that follow on the basis of the information provided in the passage.A leading Indian industrialist in a recent article on ways to strengthen India's economy has drawn attention to the problems of inflation and industrial sickness among other things.  One of the main reasons for industrial sickness in our country has been the fact that business and industrial managers, have not been able to look beyond the immediate future.  They have been too preoccupied with their attempts to report favorable results for the current year higher profits and larger dividends to the share holders.  The planning horizon has hardly ever exceeded five years. Investmentshave been inadequate for new plants and towards diversification and expansions.  Modernisation and asset creation has seriously lagged behind. In business, growth is needed for survival; one has to grow if one does not want to be wiped out This is particularly true today with liberalisation of imports and increasing competition.  More over, growth and higher productivity createemployment and higher employment creates larger markets both for industrial and consumer products.  It was Hentry Ford   who brought home the need for the creation of larger and a more stable middle class, that is, a larger number of people who can afford more and more of goods and services.  Even after forty years of independence our industrialists have not been able to shedthe petty shopkeeper's mentality and our highly educated management has tagged along merrily and without concern.21. Which of the following short comings of Indian industrialists has been highlighted by the author ?    A) The invest unreasonable high amount on diversification and expansion.    B) They are more concerned for immediate net gains than for developmental activities    C) They are reluctant to maintain the shopkeeper mentality    D) They are less concerned for payment of dividends to shareholders    E) None of the above     Ans:  B22. The leading industrialist attributes industrial sickness mainly to    A) lacunae in five-year plans    B) preoccupations of managers with matters unrelated to business    C) higher profits and larger dividends to shareholders    D) lack of foresight among managers    E) inflation and other economic problems    Ans:  D23.According to the passage, growth and increasing productivity lead to    A) imposition of restrictions on imports    B) employment and thus provide an outlet to industrial and consumer products

Page 72: Aptitude Questions

72

    C) encouragement to export of excess consumer goods    D) disproportionate surplus of commodities    E) None of the above     Ans:  B24. Why did Hentry Ford stress the need for a more stable middle class ?    A) They are mostly service oriented    B) They do not have shopkeeper mentality    C) They can afford to buy more and more expensive goods    D) They are most unstable    E) None of these     Ans:. C25. The planning horizon has hardly ever exceeded five year's implies    A) Planning should not be for a period of less than five years    B) The planning process is very time consuming    C) The planners are not inclined to think of future    D) Planning should take care of all probable ups and downs in the next five-year period    E) Five-year period is too short for successful implementation of plans.    Ans: E26. According to the passage, the net gains pursued by managers are at the cost of     A) Diversification, modernisation and asset creation    B) Availability of markets for  industrial and consumer products    C) Inflation and industrial sickness    D) Liberalisation of imports and increasing competition    E) Higher profits and larger dividends to shareholders     Ans:  ADirections 27-32: Pick out the most effective word from the given words  to fill in the blank to make the sentence meaningfully complete.27. She had just................down when the telephone rang    A) laid    B) lain    C) lay     Ans : B28. Thinking that the other candidate was more deserving for the post, I.............. in his favour    A) stood down    B) stood out    C) stood over    D) stood off     Ans : A29. This building comprises...............sixty rooms    A) off    B) to    C) with    D) of     Ans : D30. She always runs short..............money    A) of    B) by    C) in    D) to     Ans : A31. Sunita decided to set..............some time every day for prayers    A) up    B) in    C) on    D) aside     Ans : D32. The Government agreed to pay compensation.................damaged crops, land and cattle.    A) of    B) for    C) to    D) through     Ans : BSection 3: CRITICAL REASONING ( 12 questions , 30 minutes)Directions 1- 12 : Answer the questions given below the passage or statement as true, false or can't say.PASSAGE : Alphine tunnels are closed tunnels. In the past 30 yrs not even a single accident has been recorded for there is one accident in the rail road system. Even in case of a fire accident it is possible to shift the passengers into adjacent wagons and even the live fire can be detected and extinguished with in the duration of 30 min.Answer questions 1-3 based on passage 1. No accident can occur in the closed tunnels    Ans. True2. Fire is allowed to live for 30 min    Ans. False3 All the care that travel in the tunnels will be carried by rail shutters.    Ans. True

Page 73: Aptitude Questions

73

PASSAGE   : In the survey conducted in mumbai out of 63 newly married house wives not a single house wife felt that the husbands should take equal part in the household work as they felt they loose their power over their husbands. Inspite of their careers they opt to do the kitchen work themselves after coming back to home. the wives get half as much leisure time as the husbands get at the week ends.Answer questions 4-6 based on passage4.Housewives want the husbands to take part equally in the household        Ans. False

5. Wives have half as much leisure time as the husbands have        Ans. False

6. 39% of the men will work equally in the house in cleaning and washing.        Ans. FalsePASSAGE   : In the totalitarian days, the words have very much devalued. In the present day, they are becoming domestic that is the words will be much more devalued. In that days, the words will be very much effected in political area. but at present, the words came very cheap .We can say they come free at cost.Answer questions 7-9 based on passage .7. Totalitarian society words are devalued.     Ans. False8. Totalitarians will have to come much about words     Ans. True9. The art totalitarian society the words are used for the political speeches.    Ans. FalsePASSAGE : In Confucius days the technology development was less and it took weeks to communicate a message. wherein we can send it through satellite with in no time. even with this fast developments it has become difficult to understand each other. Answer questions 10-12 based on passage .10. people were not intelligent during Confucius days      Ans. False11. transport facilities are very much improved in now-a-days       Ans. True12. even with the fast developments of the technology we cannot live happily.       Ans. can't say

1)APTITUDE TEST:

Questions = 82 ; time limit = 90 minutes. no negative marking. Offline (paper & pen) test and a

PSYCHOMETRY TEST also.Section 1: VERBAL    ( 32 Questions ,20 minutes )Directions for questions 1-10:Find the synonyms of the following words

 1.     Depreciation        A. appreciation        B.  Deflation        C.  rise        D.  None of these        Ans: B2.  Circumspect          A Condition         B Inspect         C. Cautious          D Reckless         Ans: C3.  Abysmal          A. Slight            B. Deep             C. Illustrious              D. Terrible          Ans: D4.    Diligent        A. hardworking        B. delinquent        C. neglectful          D. remiss        Ans: A5.    Vehement         A. Passionate    B. Confess     C. Noisy   D Moqulis         Ans: A6.    Impetus       A. Connect      B. Crucial       C. Stimulus    D Immediate

Page 74: Aptitude Questions

74

        Ans: C7.   Acronym        A.  Abbreviation    B. Similar        C. analogous      D. correspondent          Ans: A8.   Disseminate        A.  Forecast   B. Spread   C Barns         D. unextended         Ans: B9.   Harbinger  A. Naval    B. Uncommon  C. Fore Runner    D. Glory       Ans: C10. Ponderous       A. light             B. cumbersome         C. interesting            D. None of these       Ans: CDirections for questions 11-20:Find the Antonyms of the following words11) Tractable    A. Objectionable          B. Enjoyable          C.  Adaptable           D. ObstinateAns: A12) Covert A. Manifest          B. Invisible           C. Scared            D. AlterAns: A13) Pensive  A. Repentant     B. Sad     C. Thoughtless      D. Careless Ans: C14) Mitigate  A.  Aggravate    B. Relieve    C. Eliminate      D. ExhumeAns: A15) Diverge A. Contrary             B. Coming Together         C. Conversant      D. ControversyAns: B16) Dogmatic A. Skeptical   B. Resilient   C. Stubborn    D. SuspiciousAns: D17) Clutch A. Hold     B. Grab    C. Release       D. SpreadAns: C18) MotleyA. Bulky    B. Speckled     C. Homogeneous     D. DifferentAns: C19) Relinquish A. Pursue     B. Vanquish     C. Destroy          D. DevastateAns: A20) Transient (I) Permanent             (Ii) Removed        C. ephemeral        D. passing Ans: ADirections for Questions 21-26: Read the passage and answer the questions that follow on the basis of the information provided in the passage. Nature is like business. Business sense dictates that we guard our capital and live from the interest.  Nature's capital is the enormous diversity of living things. Without it, we cannot feed ourselves, cure ourselves of illness or provide industry with the raw materials of wealth creation. Professor Edward Wilson, of Harvard University says," The folly our descendants are least likely to forgive us is the ongoing loss of genetic and spices diversity.  This will take millions of years to correct".Only 150 plant species have ever been widely cultivated.  Yet over 75000 edible plants are known in the wild. In a hungry world, with a population growing by 90 million each year, so much wasted potential in tragic. Medicines from the wild are worth around 40 billion dollars a year. Over 5000 species are known to yield chemical with cancer fighting potential Scientists currently estimate that the total number of species in the world is between 10-30 million with only around 1.4 million identified.The web of life is torn when mankind exploits natural resources in short-sighted ways. The trade in tropical hardwoods can destroy whole forests to extract just a few commercially attractive specimens.  Bad agricultural practice triaggers 24 billion tonnes of top soil erosion a year losting the equivalent of 9 million tonnes of grain output.  Cutting this kind of unsuitable exploitation and instituting "sustainable utilisation" will help turn the environmental crisis around.21. Why does the author compare 'nature' to business ?    A) Because of the capital depletion in nature and business    B) Because of the similarity with which one should use both    C) Because of the same interest level yield    D) Because of the diversity of the various capital inputs. Ans : B22. "The folly our descendants are least likely to forgive us". What is the business equivalent of the folly the author is referring to ?    A) Reducing the profit margin    B) Not pumping some money out of profits into the business    C) Eroding the capital lease of the business    D) Putting interest on capital back into the business   Ans: C23. Which of the following statements in false in context of the given passage ?    A) The diversity of plant life is essential for human existence    B) Scientists know the usefulness of most plat species    C) Chemicals for cancer treatment are available from plants.     D) There are around ten times the plant species undiscovered as compared to the discovered ones  Ans: B24.Which of the following correctly reflects the opinion of the author to take care of hunger of the world ?

Page 75: Aptitude Questions

75

    A) Increase the number of the edible plants being cultivates.    B) Increase cultivation of the 150 species presently under cultivation    C) Increase the cultivation of medical plants    D) Increase the potential of the uncultivated edible plants ? Ans: D25. Which of the following is mentioned as the immediate cause for the destruction of plant species ?    A) Soil Erosion    B) Destruction of habitat    C) Cultivation    D) Agricultural practices  Ans: B26. Choose the word which is nearly same in meaning to the given word as used in the passage,.    WastedA) Consumed    B) Squandered    C) Unutilized    D) Unprofitable Ans: CDirections 27-32 : Pick out the most effective word from the given words  to fill in the blank to make the sentence meaningfully complete.27. Priya is not............. for this kind of a job    A) cut in    B) cut up    C) cut through    D) cut out Ans : D28. He left the book................ the telephone    A) around    B) beside    C) besides    D) at Ans : B29. The waiter took the plates.................after we had finished eating    A) up    B) off    C) away    D) out Ans : C30. It is fourteen years since I...............him    A) saw    B) have seen    C) did see    D) had seen Ans : A31. I have done my muddled but...................honest best    A) never the less    B) rather    C) none of these Ans : C32. It is mainly due to their lethargy that the plan fell...................    A) over    B) out    C) through    D) off Ans : CSection 2: QUANTITATIVE/LOGICAL REASONING ( 38 questions , 40 minutes )1. There are 150 weights .Some are 1 kg weights and some are 2 kg weights. The sum of the weights is 260.          What is the number of 1kg weights?Ans. 402. A is driving on a highway when the police fines him for over speeding and exceeding the limit by 10 km/hr. At the      same time B is fined for over speeding by twice the amount by which A exceeded the limit. If he was driving at     35 km/hr what is the speed limit for the road?Ans. 15 kmph3. A moves 3 kms east from his starting point . He then travels 5 kms north. From that point he moves 8 kms to the     east. How far is A from his starting point?     Ans. 13 kms4. A car travels 12 kms with a 4/5th filled tank. How far will the car travel with 1/3 filled tank?Ans. 5 kms5. The sum of the digits of a two digit number is 8. When 18 is added to the number, the digits are reversed. Find      the number?Ans. 356. The cost of one pencil, two pens and four erasers is Rs.22 while the cost of five pencils, four pens and two erasers is Rs.32.How much will three pencils, three pens and three erasers cost?Ans. 277. Fathers age is 5 times his son's age. 4 years back the father was 9 times older than son. Find the fathers' present     age.Ans. 40 years8.  What number should be added to or subtracted from each term of the ratio17 : 24  so that it becomes equal to        1 : 2  ?Ans. 10 should be subtracted9. What is the 12th term of the series 2, 5, 8, ....Ans. 3510. If 20 men take 15 days to to complete a job, in how many days can 25 men finish that work?Ans. 12 days11. In a fraction, if 1 is added to both the numerator at the denominator, the fraction becomes 1/2. If numerator is        subtracted from the denominator, the fraction becomes 3/4. Find the fraction.Ans. 3/712. If Rs.1260 is divided between between A, B and C in the ratio 2:3:4, what is C's share?Ans. Rs. 56013. A shopkeeper bought a watch for Rs.400 and sold it for Rs.500.What is his profit percentage?Ans. 25%14. What percent of 60 is 12?  Ans. 20%

Page 76: Aptitude Questions

76

15. Hansie made the following amounts in seven games of cricket in India: Rs.10, Rs.15, Rs.21, Rs.12, Rs.18,       Rs.19 and Rs.17(all figures in crores of course).Find his average earnings.Ans. Rs.16 crore16. If two pencils cost 8 cents, then how much do 5 pencils cost?Ans. 20 cents17. Some work is done by two people in 24 minutes. One of them can do this work alone in 40 minutes. How       much  time does the second person take to do the same work ? Ans. 60 minutes18. A car is filled with four and half gallons of fuel for a round trip.If the amount of fuel taken while going is 1/4       more than the amount taken for coming, what is the amount of fuel consumed while coming back?Ans.2 gallons19. The lowest temperature in the night in a city A is 1/3 more than 1/2 the highest during the day. Sum of the        lowest  temperature and the highest temperature is 100 degrees. Then what is the low temp?Ans.40 degrees20. Javagal, who decided to go to weekened trip should not exceed 8 hours driving in a day. The average speed of       forward journey is 40 miles/hr.Due to traffic on sundays, the return journey's average speed is 30 m/h. How far       he can select a picnic spot?      Ans. 120 miles21. A salesperson by mistake multiplied a number and got the answer as 3, instead of dividing the number by       3.What is the answer he should have actually got?Ans. 322. A building with height D shadow upto G. What is the height of a neighbouring building with a shadow of C feet.Ans. (C*D)/G23. A person was fined for exceeding the speed limit by 10 mph. Another person was also fined for exceeding the      same speed limit by twice the same. If the second person was travelling at a speed of 35 mph, find the speed       limit.  Ans. 15 mph24 A bus started from bus stand at 8.00am, and after staying for 30 minutes at a destination, it returned back to the       bus stand. The destination is 27 miles from the bus stand. The speed of the bus is 18mph. During the return       journey bus travels with 50% faster speed. At what time does it return to the bus stand?Ans. 11.00am25. In a mixture, R is 2 parts and S is 1 part. In order to make S to 25% of the mixture, how much of R is to be        added?Ans. One part of R26. Wind flows 160 miles in 330 min, for travelling 80 miles how much time does it require?Ans. 2 hrs 45 mins27. With a 4/5 full tank a vehicle can travel 12 miles, how far can it travel with a 1/3 full tank        Ans. 5 miles28. There are two trees in a lawn. One grows at a rate 3/5 of the other in 4 years. If the total growth of trees is 8 ft.       What is the height of the smaller tree after 2 yearsAns. 1 1/2 feet29. Refer to the figure below. A ship started from P and moves at a speed of I miles per hour and another ship        starts from L and moving with H miles per hour simultaneously .Where do the two ships meet?       ||---g---||---h---||---i---||---j---||---k---||---l---||       PG H I J K L are the various stops in between denoted by || . The values g, h, i, j, k, l denote the distance        between the ports. Ans. Between I and J, closer to J30. If A is travelling at 72 km per hour on a highway. B is travelling at a speed of 25 meters per second on a       highway. What is the difference in their speeds in m/sec.Ans. 1 m/sec31. If the word 'ddosszm' is changed to 'central' then what will be the change for 'rtjbl' ?Ans:quick

Page 77: Aptitude Questions

77

32. The word unimpressive was given.they asked us to do change 1st & 2nd,3rd & 4th,so on.then they asked what       will be 10th letter from right?Ans: m33. The ques on a man,a woman and a boy finish work together in 6 days.man takes 10 days,woman takes 24 days       then how much boy will take? Ans:40 days34. If  DDMUQZM is coded as CENTRAL then RBDJK can be coded as ---------Ans. QCEIL35. In the word ECONOMETRICS, if the first and second , third and forth ,forth and fifth, fifth and sixth words are       interchanged up to the last letter, what would be the tenth letter from right?        Ans. word is CENOMOTEIRSC tenth word is R

36.Find the physical quantity in units from the equation: (Force*Distance)/(Velocity*Velocity)      Ans. Ns2/m37. Find the value of @@+25-++@16, where @ denotes "square" and + denotes "square root".  Ans: 62138. If f(0)=1 and f(n)= f(n-1)*n, find the value of f(4).Ans: 24Section 3: CRITICAL REASONING ( 12 questions , 30 minutes)Directions 1-12: Answer the questions given below the passage or statement as true, false or can't say.PASSAGE : My father has no brothers. He has three sisters who has two child's each.Answer 1-5 based on the passage 1. My grandfather has two sons .Ans. False   2. Three of my aunts have two sons    Ans. Can't say3. My father is only child to his father    Ans. False4. I have six cousins from my mother side    Ans. Can't say5. I have one uncle   Ans. Can't say (uncle can be from the mother's side as well)PASSAGE: Ether injected into gallbladder to dissolve cholesterol based gallstones. This type one day treatment is enough for gallstones not for calcium stones. This method is alternative to surgery for millions of people who are suffering from this disease. Answer questions 6-9 based on passage6.Calcium stones can be cured in one day  Ans. False 7. Hundreds of people contains calcium stone Ans. Can't say8. Surgery is the only treatment to calcium stonesAns. True9. Ether will be injected into the gallbladder to cure the cholesterol based gall stones    Ans. TruePASSAGE : Hacking is illegal entry into another computer. This happens mostly because of lack of knowledge of computer networking. With networks one machine can access to another machine. Hacking go about without knowing that each network is accredited to use network facility.Answer questions 10-12 based on passage 10. Hackers never break the code of the company which they work for        Ans. Can't say

11. Hacking is the only vulnerability of the computers for the usage  of the data Ans. False12.Hacking is done mostly due to the lack of computer knowledge        Ans. False